Any open subset of $Bbb R$ is a at most countable union of disjoint open intervals. [Collecting Proofs]












150












$begingroup$


This question has probably been asked. However, I am not interested in just getting the answer to it. Rather, I am interested in collecting as many different proofs of it which are as diverse as possible. A professor told me that there are many. So, I invite everyone who has seen proofs of this fact to share them with the community. I think it is a result worth knowing how to prove in many different ways and having a post that combines as many of them as possible will, no doubt, be quite useful. After two days, I will place a bounty on this question to attract as many people as possible. Of course, any comments, corrections, suggestions, links to papers/notes etc. are more than welcome.










share|cite|improve this question











$endgroup$








  • 57




    $begingroup$
    First proof that comes in mind: if $O$ is an open set and $x in O$ then there exists an interval $I$ such that $x in I subset O$. If there exists one such interval, then there exists one 'largest' interval which contains $x$ (the union of all such intervals). Denote by ${I_alpha}$ the family of all such maximal intervals. First all intervals $I_alpha$ are pairwise disjoint (otherwise they wouldn't be maximal) and every interval contains a rational number, and therefore there can only be a countable number of intervals in the family.
    $endgroup$
    – Beni Bogosel
    Mar 2 '13 at 0:12








  • 1




    $begingroup$
    Since this is a big-list question, I am converting it to CW.
    $endgroup$
    – robjohn
    Mar 2 '13 at 9:01










  • $begingroup$
    Oh, OK! Thanks!
    $endgroup$
    – Orest Xherija
    Mar 3 '13 at 1:04










  • $begingroup$
    I cannot understand how each $I_a$ is disjoint. If one interval contains x and another interval also contains x, aren't they intersecting?
    $endgroup$
    – edgaralienfoe
    May 29 '14 at 1:41












  • $begingroup$
    Yes but their union would then be another interval that contains an $I_a$ and therefore must be equal to it by maximality.
    $endgroup$
    – Gregory Grant
    Mar 15 '15 at 16:36
















150












$begingroup$


This question has probably been asked. However, I am not interested in just getting the answer to it. Rather, I am interested in collecting as many different proofs of it which are as diverse as possible. A professor told me that there are many. So, I invite everyone who has seen proofs of this fact to share them with the community. I think it is a result worth knowing how to prove in many different ways and having a post that combines as many of them as possible will, no doubt, be quite useful. After two days, I will place a bounty on this question to attract as many people as possible. Of course, any comments, corrections, suggestions, links to papers/notes etc. are more than welcome.










share|cite|improve this question











$endgroup$








  • 57




    $begingroup$
    First proof that comes in mind: if $O$ is an open set and $x in O$ then there exists an interval $I$ such that $x in I subset O$. If there exists one such interval, then there exists one 'largest' interval which contains $x$ (the union of all such intervals). Denote by ${I_alpha}$ the family of all such maximal intervals. First all intervals $I_alpha$ are pairwise disjoint (otherwise they wouldn't be maximal) and every interval contains a rational number, and therefore there can only be a countable number of intervals in the family.
    $endgroup$
    – Beni Bogosel
    Mar 2 '13 at 0:12








  • 1




    $begingroup$
    Since this is a big-list question, I am converting it to CW.
    $endgroup$
    – robjohn
    Mar 2 '13 at 9:01










  • $begingroup$
    Oh, OK! Thanks!
    $endgroup$
    – Orest Xherija
    Mar 3 '13 at 1:04










  • $begingroup$
    I cannot understand how each $I_a$ is disjoint. If one interval contains x and another interval also contains x, aren't they intersecting?
    $endgroup$
    – edgaralienfoe
    May 29 '14 at 1:41












  • $begingroup$
    Yes but their union would then be another interval that contains an $I_a$ and therefore must be equal to it by maximality.
    $endgroup$
    – Gregory Grant
    Mar 15 '15 at 16:36














150












150








150


122



$begingroup$


This question has probably been asked. However, I am not interested in just getting the answer to it. Rather, I am interested in collecting as many different proofs of it which are as diverse as possible. A professor told me that there are many. So, I invite everyone who has seen proofs of this fact to share them with the community. I think it is a result worth knowing how to prove in many different ways and having a post that combines as many of them as possible will, no doubt, be quite useful. After two days, I will place a bounty on this question to attract as many people as possible. Of course, any comments, corrections, suggestions, links to papers/notes etc. are more than welcome.










share|cite|improve this question











$endgroup$




This question has probably been asked. However, I am not interested in just getting the answer to it. Rather, I am interested in collecting as many different proofs of it which are as diverse as possible. A professor told me that there are many. So, I invite everyone who has seen proofs of this fact to share them with the community. I think it is a result worth knowing how to prove in many different ways and having a post that combines as many of them as possible will, no doubt, be quite useful. After two days, I will place a bounty on this question to attract as many people as possible. Of course, any comments, corrections, suggestions, links to papers/notes etc. are more than welcome.







real-analysis general-topology analysis big-list faq






share|cite|improve this question















share|cite|improve this question













share|cite|improve this question




share|cite|improve this question








edited 2 days ago


























community wiki





4 revs, 4 users 100%
Orest Xherija









  • 57




    $begingroup$
    First proof that comes in mind: if $O$ is an open set and $x in O$ then there exists an interval $I$ such that $x in I subset O$. If there exists one such interval, then there exists one 'largest' interval which contains $x$ (the union of all such intervals). Denote by ${I_alpha}$ the family of all such maximal intervals. First all intervals $I_alpha$ are pairwise disjoint (otherwise they wouldn't be maximal) and every interval contains a rational number, and therefore there can only be a countable number of intervals in the family.
    $endgroup$
    – Beni Bogosel
    Mar 2 '13 at 0:12








  • 1




    $begingroup$
    Since this is a big-list question, I am converting it to CW.
    $endgroup$
    – robjohn
    Mar 2 '13 at 9:01










  • $begingroup$
    Oh, OK! Thanks!
    $endgroup$
    – Orest Xherija
    Mar 3 '13 at 1:04










  • $begingroup$
    I cannot understand how each $I_a$ is disjoint. If one interval contains x and another interval also contains x, aren't they intersecting?
    $endgroup$
    – edgaralienfoe
    May 29 '14 at 1:41












  • $begingroup$
    Yes but their union would then be another interval that contains an $I_a$ and therefore must be equal to it by maximality.
    $endgroup$
    – Gregory Grant
    Mar 15 '15 at 16:36














  • 57




    $begingroup$
    First proof that comes in mind: if $O$ is an open set and $x in O$ then there exists an interval $I$ such that $x in I subset O$. If there exists one such interval, then there exists one 'largest' interval which contains $x$ (the union of all such intervals). Denote by ${I_alpha}$ the family of all such maximal intervals. First all intervals $I_alpha$ are pairwise disjoint (otherwise they wouldn't be maximal) and every interval contains a rational number, and therefore there can only be a countable number of intervals in the family.
    $endgroup$
    – Beni Bogosel
    Mar 2 '13 at 0:12








  • 1




    $begingroup$
    Since this is a big-list question, I am converting it to CW.
    $endgroup$
    – robjohn
    Mar 2 '13 at 9:01










  • $begingroup$
    Oh, OK! Thanks!
    $endgroup$
    – Orest Xherija
    Mar 3 '13 at 1:04










  • $begingroup$
    I cannot understand how each $I_a$ is disjoint. If one interval contains x and another interval also contains x, aren't they intersecting?
    $endgroup$
    – edgaralienfoe
    May 29 '14 at 1:41












  • $begingroup$
    Yes but their union would then be another interval that contains an $I_a$ and therefore must be equal to it by maximality.
    $endgroup$
    – Gregory Grant
    Mar 15 '15 at 16:36








57




57




$begingroup$
First proof that comes in mind: if $O$ is an open set and $x in O$ then there exists an interval $I$ such that $x in I subset O$. If there exists one such interval, then there exists one 'largest' interval which contains $x$ (the union of all such intervals). Denote by ${I_alpha}$ the family of all such maximal intervals. First all intervals $I_alpha$ are pairwise disjoint (otherwise they wouldn't be maximal) and every interval contains a rational number, and therefore there can only be a countable number of intervals in the family.
$endgroup$
– Beni Bogosel
Mar 2 '13 at 0:12






$begingroup$
First proof that comes in mind: if $O$ is an open set and $x in O$ then there exists an interval $I$ such that $x in I subset O$. If there exists one such interval, then there exists one 'largest' interval which contains $x$ (the union of all such intervals). Denote by ${I_alpha}$ the family of all such maximal intervals. First all intervals $I_alpha$ are pairwise disjoint (otherwise they wouldn't be maximal) and every interval contains a rational number, and therefore there can only be a countable number of intervals in the family.
$endgroup$
– Beni Bogosel
Mar 2 '13 at 0:12






1




1




$begingroup$
Since this is a big-list question, I am converting it to CW.
$endgroup$
– robjohn
Mar 2 '13 at 9:01




$begingroup$
Since this is a big-list question, I am converting it to CW.
$endgroup$
– robjohn
Mar 2 '13 at 9:01












$begingroup$
Oh, OK! Thanks!
$endgroup$
– Orest Xherija
Mar 3 '13 at 1:04




$begingroup$
Oh, OK! Thanks!
$endgroup$
– Orest Xherija
Mar 3 '13 at 1:04












$begingroup$
I cannot understand how each $I_a$ is disjoint. If one interval contains x and another interval also contains x, aren't they intersecting?
$endgroup$
– edgaralienfoe
May 29 '14 at 1:41






$begingroup$
I cannot understand how each $I_a$ is disjoint. If one interval contains x and another interval also contains x, aren't they intersecting?
$endgroup$
– edgaralienfoe
May 29 '14 at 1:41














$begingroup$
Yes but their union would then be another interval that contains an $I_a$ and therefore must be equal to it by maximality.
$endgroup$
– Gregory Grant
Mar 15 '15 at 16:36




$begingroup$
Yes but their union would then be another interval that contains an $I_a$ and therefore must be equal to it by maximality.
$endgroup$
– Gregory Grant
Mar 15 '15 at 16:36










16 Answers
16






active

oldest

votes


















100





+100







$begingroup$

Here’s one to get things started.



Let $U$ be a non-empty open subset of $Bbb R$. For $x,yin U$ define $xsim y$ iff $big[min{x,y},max{x,y}big]subseteq U$. It’s easily checked that $sim$ is an equivalence relation on $U$ whose equivalence classes are pairwise disjoint open intervals in $Bbb R$. (The term interval here includes unbounded intervals, i.e., rays.) Let $mathscr{I}$ be the set of $sim$-classes. Clearly $U=bigcup_{I in mathscr{I}} I$. For each $Iinmathscr{I}$ choose a rational $q_Iin I$; the map $mathscr{I}toBbb Q:Imapsto q_I$ is injective, so $mathscr{I}$ is countable.



A variant of the same basic idea is to let $mathscr{I}$ be the set of open intervals that are subsets of $U$. For $I,Jinmathscr{I}$ define $Isim J$ iff there are $I_0=I,I_1,dots,I_n=Jinmathscr{I}$ such that $I_kcap I_{k+1}nevarnothing$ for $k=0,dots,n-1$. Then $sim$ is an equivalence relation on $mathscr{I}$. For $Iinmathscr{I}$ let $[I]$ be the $sim$-class of $I$. Then $left{bigcup[I]:Iinmathscr{I}right}$ is a decomposition of $U$ into pairwise disjoint open intervals.



Both of these arguments generalize to any LOTS (= Linearly Ordered Topological Space), i.e., any linearly ordered set $langle X,lerangle$ with the topology generated by the subbase of open rays $(leftarrow,x)$ and $(x,to)$: if $U$ is a non-empty open subset of $X$, then $U$ is the union of a family of pairwise disjoint open intervals. In general the family need not be countable, of course.






share|cite|improve this answer











$endgroup$









  • 10




    $begingroup$
    I like this answer LOTS. :-) And +1.
    $endgroup$
    – coffeemath
    Mar 2 '13 at 1:30










  • $begingroup$
    This answer is very clear. It depends on the axiom of choice though, are there any constructive variants of this argument?
    $endgroup$
    – Bunder
    Mar 2 '13 at 8:28






  • 6




    $begingroup$
    @Bunder: No, it doesn’t depend on the axiom of choice. The only place where you might even think that it did is when I showed that there are only countably many intervals, but the rationals can be explicitly well-ordered, so no choice is necessary even there.
    $endgroup$
    – Brian M. Scott
    Mar 2 '13 at 12:24










  • $begingroup$
    Is it correct to write each equivalence class as $I_x = (inf{y in U: y sim x}, sup{y in U: y sim x})$?
    $endgroup$
    – David
    Nov 4 '15 at 16:09








  • 1




    $begingroup$
    @IntegrateThis. Let $S$ be a $sim$-equivalence class. For any $xin S$ there exists $r>0$ such that $(-r+x,r+x)subset U$ so $forall yin (-r+x,r+x);(ysim x). $ So $(-r+x,r+x)subset {yin U:ysim x}=S.$
    $endgroup$
    – DanielWainfleet
    Sep 24 '18 at 20:38





















30












$begingroup$

These answers all seem to be variations on one another, but I've found each one so far to be at least a little cryptic. Here's my version/adaptation.



Let $U subseteq R$ be open and let $x in U$. Either $x$ is rational or irrational. If $x$ is rational, define
begin{align}I_x = bigcuplimits_{substack{Itext{ an open interval} \ x~in~I~subseteq~U}} I,end{align}
which, as a union of non-disjoint open intervals (each $I$ contains $x$), is an open interval subset to $U$. If $x$ is irrational, by openness of $U$ there is $varepsilon > 0$ such that $(x - varepsilon, x + varepsilon) subseteq U$, and there exists rational $y in (x - varepsilon, x + varepsilon) subseteq I_y$ (by the definition of $I_y$). Hence $x in I_y$. So any $x in U$ is in $I_q$ for some $q in U cap mathbb{Q}$, and so
begin{align}U subseteq bigcuplimits_{q~in~U cap~mathbb{Q}} I_q.end{align}
But $I_q subseteq U$ for each $q in U cap mathbb{Q}$; thus
begin{align}U = bigcuplimits_{q~in~U cap~mathbb{Q}} I_q, end{align}
which is a countable union of open intervals.






share|cite|improve this answer











$endgroup$













  • $begingroup$
    He wanted a countable union of disjoint open intervals. If $U$ is simply an open interval, then $U=I_q=I_{tilde{q}}$ for all $q,tilde{q} in mathcal{Q}cap U$.
    $endgroup$
    – José Siqueira
    Oct 15 '13 at 16:15








  • 2




    $begingroup$
    That it is a disjoint union follows from the definition of $I_q$: if $x in I_q cap I_p$ then $I_q cup I_p subseteq I_q$ and $I_p$; hence if $I_q neq I_p$ then $I_q cap I_p = emptyset$. Strictly speaking one should throw away all repeated $I_q$s, and one can definitely do this without destroying the countability of the union.
    $endgroup$
    – Stromael
    Oct 21 '13 at 11:14



















15












$begingroup$

In a locally connected space $X$, all connected components of open sets are open. This is in fact equivalent to being locally connected.



Proof: (one direction) let $O$ be an open subset of a locally connected space $X$. Let $C$ be a component of $O$ (as a (sub)space in its own right). Let $x in C$. Then let $U_x$ be a connected neighbourhood of $x$ in $X$ such that $U_x subset O$, which can be done as $O$ is open and the connected neighbourhoods form a local base. Then $U_x,C subset O$ are both connected and intersect (in $x$) so their union $U_x cup C subset O$ is a connected subset of $O$ containing $x$, so by maximality of components $U_x cup C subset C$. But then $U_x$ witnesses that $x$ is an interior point of $C$, and this shows all points of $C$ are interior points, hence $C$ is open (in either $X$ or $O$, that's equivalent).



Now $mathbb{R}$ is locally connected (open intervals form a local base of connected sets) and so every open set if a disjoint union of its components, which are open connected subsets of $mathbb{R}$, hence are open intervals (potentially of infinite "length", i.e. segments). That there are countably many of them at most, follows from the already given "rational in every interval" argument.






share|cite|improve this answer











$endgroup$













  • $begingroup$
    "connected => interval" also requires a proof (usually using the intermediate step "path-connected").
    $endgroup$
    – Martin Brandenburg
    Mar 4 '13 at 19:29






  • 1




    $begingroup$
    Path connected is not needed. It would be circular, as one needs connectedness of intervals to see that path-connected implies connected... If a set is not order convex (so x < z < y, x,y in A but z not in A), then we have an immediate disconnection. That intervals are connected follows from completeness of the order, and is a standard fact for ordered spaces.
    $endgroup$
    – Henno Brandsma
    Mar 4 '13 at 20:22



















11












$begingroup$

Let $Usubseteqmathbb R$ open. Is enough to write $U$ as a disjoint union of open intervals.

For each $xin U$ we define $alpha_x=inf{alphainmathbb R:(alpha,x+epsilon)subseteq U, text{ for some }epsilon>0}$ and $beta_x=sup{betainmathbb R:(alpha_x,beta)subseteq U}$.



Then $displaystyle U=bigcup_{xin U}(alpha_x,beta_x)$ where ${(alpha_x,beta_x):xin U}$ is a disjoint family of open intervals.






share|cite|improve this answer











$endgroup$









  • 1




    $begingroup$
    Are you sure the union is countable here?
    $endgroup$
    – user10444
    Sep 11 '14 at 16:55






  • 1




    $begingroup$
    @user10444: It is countable yes! If you agree that ${(alpha_x,beta_x):xin U}$ is a disjoint family of open intervals then you can see it by choosing $r_xin mathbb Qcap (alpha_x,beta_x)$ for all $xin U$. Then ${r_x:xin U}$ is countable right? Note that the intervals ${(alpha_x,beta_x):xin U}$ are not distinct!
    $endgroup$
    – P..
    Sep 11 '14 at 19:03








  • 1




    $begingroup$
    Recall, we can "choose" such an $r_{x}in(alpha_{x},beta_{x})$ such that $r_{x}inmathbb{Q}$ by the Axiom of Choice and by the density of $mathbb{Q}$ in $mathbb{R}$.
    $endgroup$
    – Procore
    Aug 30 '17 at 20:29





















6












$begingroup$

A variant of the usual proof with the equivalence relation, which trades in the ease of constructing the intervals with the ease of proving countability (not that either is hard...):




  1. Define the same equivalence relation, but only on $mathbb Q cap U$:
    $q_1 sim q_2$ iff $(q_1, q_2) subset U$ (or $(q_2, q_1) subset U$, whichever makes sense).

  2. From each equivalency class $C$, produce the open interval $(inf C, sup C) subset U$ (where $inf C$ is defined to be $-infty$ in case $C$ is not bounded from below, and $sup C = infty$ in case $C$ is not bounded from above).

  3. The amount of equivalence classes is clearly countable, since $mathbb Q cap U$ is countable.






share|cite|improve this answer











$endgroup$





















    6












    $begingroup$

    Let $U$ be an open subset of $mathbb{R}$. Let $P$ be the poset consisting of collections $mathcal{A}$ of disjoint open intervals where we say $mathcal{A} le mathcal{A}'$ if each of the sets in $mathcal{A}$ is a subset of some open interval in $mathcal{A}'$. Every chain $C$ in this poset has an upper bound, namely $$mathcal{B} = left{ bigcupleft{J in bigcupbigcup C : I subseteq J right}: I in bigcupbigcup Cright}.$$
    Therefore by Zorn's lemma the poset $P$ has a maximal element $mathcal{M}$. We claim that the union of the intervals in $mathcal{M}$ is all of $U$. Suppose toward a contradiction that there is a real $x in U$ that is not contained in any of the intervals in $mathcal{M}$. Because $U$ is open we can take an open interval $I$ with $x in I subseteq U$.
    Then the set
    $$mathcal{M}' = {J in mathcal{M} : J cap I = emptyset} cup left{I cup bigcup {J in mathcal{M} : J cap I ne emptyset}right}$$
    is a collection of disjoint open intervals and
    is above $mathcal{M}$ in the poset $P$, contradicting the maximality of $mathcal{M}$. It remains to observe that $mathcal{M}$ is countable, which follows from the fact that its elements contain distinct rational numbers.



    Note that the only way in which anything about order (or connectedness) is used is to see that $I cup bigcup {J in mathcal{M} : J cap I ne emptyset}$ is an interval.






    share|cite|improve this answer











    $endgroup$













    • $begingroup$
      And yes, before you ask, I know this proof is silly.
      $endgroup$
      – Trevor Wilson
      Mar 8 '13 at 2:21






    • 2




      $begingroup$
      I think this proof conceptually connects many things, including well-ordering, order theory and so on. So, I really benefited.
      $endgroup$
      – user64066
      Oct 22 '13 at 20:19










    • $begingroup$
      @user64066 Thanks, I'm glad to hear it.
      $endgroup$
      – Trevor Wilson
      Oct 22 '13 at 20:30



















    4












    $begingroup$

    Let $G$ be a nonempty open set in $mathbb{R}$. Write $asim b$ if the closed interval $[a, b]$ or $[b, a]$ if $b<a$, lies in $G$.This is an equivalence relation, in particular $asim a$ since ${a}$ is itself a closed interval. $G$ is therefore the union of disjoint equivalence classes.



    Let $C(a)$ be the equivalence class containing $a$. Then $C(a)$ is clearly an interval. Also $C(a)$ is open, for if $kin C(a)$, then $(k-epsilon, k+epsilon)subseteq G$ for sfficiently small $epsilon$.



    But then $(k-epsilon, k+epsilon)subseteq C(a)$; so $G$ is the union of disjoint intervals. These are at most countable in number by Lindel$ddot{rm o}$f's theorem. This completes the proof.



    Reference:G. De Barra, Measure theory and Integration, Horwood Publishing.



    (http://infoedu.ir/wp-content/uploads/2014/03/MeasureTheoryBook.pdf)






    share|cite|improve this answer











    $endgroup$





















      4












      $begingroup$

      $mathbb{R}$ with standard topology is second-countable space.



      For a second-countable space with a (not necessarily countable) base, any open set can be written as a countable union of basic open set.



      Given any base for a second countable space, is every open set the countable union of basic open sets?



      Clearly, collection of open intervals is a base for the standard topology.
      Hence any open set in $mathbb{R}$ can be written as countable union of open intervals.



      If any two of exploited open intervals overlap, merge them. Then we have disjoint union of open intervals, which is still countable.






      share|cite|improve this answer











      $endgroup$





















        4












        $begingroup$

        This proof is an extended version of the nice proof proposed by
        Stromael and it serves best for beginners who want to understand every detail(that one that for any established mathematician logically seems trivial) of the proof.



        $ textbf{Proof:} $



        Let $U subseteq R$ be open and let $x in U$. Then Either $x$ is rational or $x$ is irrational.



        Suppose $x$ is rational, then define



        begin{align} I_x = bigcuplimits_{substack{Itext{ an open interval} \ x~in~I~subseteq~U}} I,end{align}



        Claim: $I_x$ is interval, $I_x$ is open and $ I_x subseteq U $



        Definition: An interval is a subset $ I subseteq R$ such that, for all $ a<c<b$ in $R$, if $ a,b in I $ then $ c in I$.



        Now, consider any $ a<c<b $ such that $ a,b in I_x$. We want to show that $ c in I_x $.



        Denote $I_a $ to be an interval such that $ x in I_a $ and $ a in I_a $. In other words $ I_a $ is one of the intervals from the union $ I_x $ that contains $a$. In the same way, let $ I_b $ be the interval such that $ x in I_b $ and $ b in I_b $.




        1. $ c=x $: If $c=x$ then by construction of $I_x$, $ x in I_x$


        2. $ c<x $: If $c<x$ then we have that either $ a<c<x<b $ or $ a<c<b<x $. Since $ x in I $ for every open interval $I$ of the union $I_x$ (by construction of $I_x$ ), we have that $x in I_a $ and $ x in I_b$. Since $ x in I_a $ then because $ I_a $ is an interval $ c in I_a$ and hence $ c in I_x $. And since $ x in I_b $ then because $ I_b $ is an interval $ c in I_b $ and hence $ c in I_x $. Thus, we concluded that $ c in I_x $.


        3. $ c > x $: If $ c>x $ then we have that either $ a<x<c<b $ or $ x<a<c<b $. Since $ x in I $ for every open interval $I$ of the union $I_x$ (by construction of $I_x$ ), we have that $x in I_a $ and $ x in I_b$. Since $ x in I_b $ then because $ I_b $ is an interval $ c in I_b $ and hence $ c in I_x $. As for the second case, note that since $ x in I_b$ we have that $ a in I_b $. But then, because $ I_b $ is an interval we have that $ c in I_b $ and hence $ c in I_x$. Hence we concluded that $ c in I_x $.



        This Proves that $ I_x $ is an interval.



        $ I_x $ is open because it is union of open sets.



        $ I_x subseteq U $ by construction.



        Suppose $x$ is irrational, then by openness of $ U $ there is $varepsilon > 0$ such that $(x - varepsilon, x + varepsilon) subseteq U$, and by the property of real numbers that for any irrational number there exists a sequence of rational unmbers that converges to that irrational number, there exists rational $y in (x - varepsilon, x + varepsilon) $. Then by construction $ (x - varepsilon, x + varepsilon) subseteq I_y $. Hence $x in I_y$. So any $x in U$ is in $I_q$ for some $q in U cap mathbb{Q}$, and so



        begin{align}U subseteq bigcuplimits_{q~in~U cap~mathbb{Q}} I_q.end{align}



        But $I_q subseteq U$ for each $q in U cap mathbb{Q}$; thus



        begin{align}U = bigcuplimits_{q~in~U cap~mathbb{Q}} I_q, end{align}



        which is a countable union of open intervals.



        Now let's show that intervals $ {I_q } ~ q in U cap mathbb{Q} $ are disjoint. Suppose there is $ i, j, in U cap mathbb{Q} $ such that $ I_i cap I_j neq emptyset $ then $ I_i subseteq I_q $ and $ I_j subseteq I_q $ for some $ q in U cap mathbb{Q} $



        Hence we constructed disjoint intervals $ {I_q } ~ q in U cap mathbb{Q} $ that are enumerated by rational numbers in $U$ and whose union is $U$. Since any subset of rational numbers is countable, $ {I_q } ~ q in U cap mathbb{Q} $ is countable as well. This finishes the proof.






        share|cite|improve this answer











        $endgroup$









        • 1




          $begingroup$
          What is the guarantee of the statement 'Denote $I_a $ to be an interval such that $ x in I_a $ '?
          $endgroup$
          – Unknown x
          Dec 30 '18 at 14:47










        • $begingroup$
          Also to me the first part of this answer seemed obscure, I tried to clarify it here math.stackexchange.com/questions/3040319/…
          $endgroup$
          – Jack J.
          Jan 17 at 9:28



















        3












        $begingroup$

        I hope that this is right as this is a lemma i've thought of and i plan to use in a project due in several days and it somewhat generalizes the question asked:



        Suppose that $U$ is a set of intervals in $mathbb{R}$ (closed, open, semi-closed, etc.). Then there exists a set of disjoint intervals $V$ in $mathbb{R}$ s.t. $bigcup_{Iin U}I=biguplus_{Iin V}I$. If non of the intervals are degenerate or $U$ is countable, then this set can be taken to be countable. And if they were all open, then we could take the segments of $V$ to be open (And also, if $U$ is countable then we won't be needing the Axiom of Choice).



        proof:
        Let us order the elements of $U$: $U=langle I_beta,|,betaleqalpharangle$ where $alpha$ is the first ordinal of cardinality $|U|$. (If $U$ is countable then this doesn't require AC and from here on will be standard induction with a simple construction in the end for $omega$).



        I'll build $V_beta=langle J^gamma_beta,|,gammaleqbetarangle$ - a sequence of segments for all $betaleqalpha$ such that every two sets in $V_beta$ are either disjoint or equal and such that $displaystyle{bigcup_{gammaleqbeta}I_gamma=biguplus_{gammaleqbeta}J^gamma_beta}$ and $forallbeta$, $langle J^beta_gammarangle_{gammageqbeta}$ is a non-descending sequence of sets ,by means of transfinite induction.



        For $V_0$ take, $V_0=langle I_0rangle$. Suppose that we have built the required $V_gamma$, $gamma<beta$ for some $betaleqalpha$, then we will build $V_beta$ in the following way: $forallgamma<beta$, denote $widetilde{J}_gamma$=$bigcup_{gammaleqdelta<beta}J_delta^gamma$-still segments (non-decreasing sequence). If $I_beta$ is disjoint of all $widetilde{J}_gamma$, taking $V_beta!=!langle widetilde{J}_gamma,|,gamma<betaranglecup{(beta,I_beta)}$ would give us a sequence $langle V_gamma,|,gammaleqbetarangle$ satisfying the required conditions of it (the only non trivial thing is that pairs of $widetilde{J}_gamma$ are either disjoint of each other or are equal, but that is also quite trivial since if the contrary would have occurred, then $existsgamma_1<gamma_2<beta$ s.t. $widetilde{J}_{gamma_1}neqwidetilde{J}_{gamma_2}$ and $widetilde{J}_{gamma_1}capwidetilde{J}_{gamma_2}neqemptyset$, but then, $exists beta>delta_1geqgamma_1, beta>delta_2geqgamma_2$ s.t. $J^{gamma_1}_{delta_1}cap J^{gamma_2}_{delta_2}neqemptyset$, meaning that either $J^{gamma_1}_{delta_2}= J^{gamma_2}_{delta_2}$ or $J^{gamma_1}_{delta_1}= J^{gamma_2}_{delta_1}$ thus, $forallbeta>epsilongeqdelta_1,delta_2$, $J^{gamma_1}_{epsilon}= J^{gamma_2}_{epsilon}$ and since we are talking here about non-decreasing sequences, this will contradict $widetilde{J}_{gamma_1}neqwidetilde{J}_{gamma_2}$). And if $I_beta$ isn't disjoint of all $widetilde{J}_gamma$, Then we can take $J_beta^gamma=widetilde{J}_gamma$ for all $gamma<beta$ that don't intersect with $I_beta$ and $J_beta^gamma=bigcup_{delta<betatext{ s.t. }widetilde{J}_deltacap I_betaneqemptyset}{widetilde{J}_delta}cup I_beta$ - segment for all of the other $gammaleqbeta$. Then again from the same arguments, $langle V_gamma,|,gammaleqbetarangle$ would satisfy the required conditions.



        Finally, we can take $V={J_alpha^beta,|,betaleqalpha}$ to get what we wanted in the first place. And obviously, if our segments were all non-degenerate to begin with, from the way we constructed our set, all of the segments in $V$ will be non-degenerate (and thus of positive measure), but they are disjoint and so there is only a countable number of them. And if the segments in $U$ were all open, then obviously, so will the segments in $V$.$square$






        share|cite|improve this answer











        $endgroup$





















          3












          $begingroup$

          The balls with radii $frac{1}{n}$ and center at a rational number form a basis for the euclidean topology. This family is countable as it is a countable union of countable sets. We have found a countable basis, so we are done.






          share|cite|improve this answer











          $endgroup$









          • 3




            $begingroup$
            Ah, but the intervals are requested to be disjoint, and your intervals aren’t disjoint.
            $endgroup$
            – Lubin
            Oct 8 '16 at 3:11



















          2












          $begingroup$

          Essentially nothing differs here from the two previous responses which rely principally on the fact that $mathbb{R} $ is locally connected. However I present here a proof that hopefully will feel accessible to readers with a slightly lower level of topological literary at the cost of appearing cumbersome to an expert.



          Consider the connected components of $U$.
          $U_x subseteq U$ is defined to be the connected component of U containing $x$ if $U_x$ is the largest connected subset of $U$ which contains $x$.
          Clearly by definition $U_x=U_v$ if $v in U_x$.
          Therefore if $U_a cap U_b neq varnothing$ then $U_a=U_b$.
          We see that ${U_x}_{xin U}$ is a disjoint collection.
          Also it should be clear that $bigcup limits_{xin U} U_x = U$.



          Now we show that $forall x$ $U_x$ is open.
          Let $yin U_x subseteq U$.
          Since $U$ is open there exists $epsilon>0$ such that $(y-epsilon, y+epsilon )subseteq U$.
          Sets of real numbers are connected iff they are intervals, singletons or empty.
          $(y-epsilon,y+epsilon)$ an interval hence it is connected.
          Therefore since $U_y$ is the largest connected subset of $U$ containing $y$ we must have $(y-epsilon,y+epsilon)subseteq U_y =U_x$.
          This shows that $U_x$ is open for all $x$.



          $U_x$ open and connected implies that $U_x$ must be an open interval.



          Also $mathbb{Q}$ dense in $mathbb{R}$,
          so $forall xin U$, $U_xcap mathbb{Q}neq varnothing$ and $U_x=U_q$ for some $qinmathbb{Q}$.
          So we can write ${U_x}_{xin U}={U_q}_{qin S}$ for some $Ssubseteq mathbb{Q}$.
          $mathbb{Q}$ is countable so $S$ is at most countable.



          In conclusion, we have just shown that the union of the connected components of $U$ is a disjoint union of open intervals that equals $U$ and is at most countable.






          share|cite|improve this answer











          $endgroup$





















            1












            $begingroup$

            The proof that every open set is a disjoint union of countably many open intervals relies on three facts:




            • $Bbb R$ is locally-connected

            • $Bbb R$ is ccc

            • The open connected sets in $Bbb R$ are open intervals


            Let $Usubseteq Bbb R$ be open. Then there is a collection of disjoint, open, connected sets ${G_alpha}_{alphain A}$ such that $U=bigcup_{alphain A} G_alpha$. Since $Bbb R$ is ccc, the collection ${G_alpha}$ is at most countable. Since the open connected sets $Bbb R$ are open intervals, ${G_alpha}$ is a countable collection of disjoint, open intervals.



            The first two facts allow us to see some generalizations. Namely any open set in a locally-connected, ccc space is a countable disjoint union of connected open sets. This applies to any Euclidean space. Although open connected subsets of Euclidean space are more complicated than open intervals, they are still relatively well-behaved.






            share|cite|improve this answer











            $endgroup$













            • $begingroup$
              What does "R is ccc" mean?
              $endgroup$
              – Christian Bueno
              Feb 24 '15 at 8:39










            • $begingroup$
              It means "satisfies the countable-chain condition" which really concerns itself with anti-chains. It means that every collection of disjoint open sets is at most countable. Every separable space is ccc.
              $endgroup$
              – Robert Wolfe
              Feb 25 '15 at 0:56










            • $begingroup$
              Ok thanks for clearing that up.
              $endgroup$
              – Christian Bueno
              Feb 25 '15 at 5:18










            • $begingroup$
              how to decompose (0,1} into a countable union of disjoint open intervals?
              $endgroup$
              – Bear and bunny
              Mar 27 '15 at 15:31



















            1












            $begingroup$

            The following is certainly not the quickest approach to a proof, but when this question was first posed to me in class, my first intuition was to use some elementary graph theory:





            Let $U$ be an open set of $mathbb{R}$. As we know, $mathbb{R}$ has a countable basis $mathcal{B}$ comprised of connected open sets and so we may write $U=bigcup_{nin I} U_n$, where for each $n$ we have $U_ninmathcal{B}$ and $I$ is some countable index set.



            Let $G$ be the intersection graph of ${U_n}$. That is to say, the vertex set of $G$ is simply ${U_n}$ and there is an edge between $U_i$ and $U_j$ iff they have nonempty intersection. It's easy to convince yourself that:




            • This graph must have countably many graphically-connected components (otherwise we'd have uncountably many vertices which is impossible).

            • The intersection graph of $Asubseteq{U_n}$ is graphically-connected iff for any two $V,Win A$ there is a sequence $V=U_{n_1},U_{n_2},ldots,U_{n_k}=W$ such that $U_{n_i}cap U_{n_{i+1}}neqvarnothing$.

            • The union $bigcup A$ is a connected set of $mathbb{R}$ whenever the intersection graph of $A$ is graphically-connected.


            Thus, when we take the union of all the vertices within a graphically-connected component, for every component, we obtain countably-many connected open sets. The union of these sets is of course $U$ itself. Since the connected open sets of $mathbb{R}$ are intervals (including rays), we're done.





            Side Note: This would also work in $mathbb{R}^n$ or in general, any topological space $X$ that has a countable basis comprised of connected sets. Well, so long as we replace countable union of disjoint open intervals with countable union of disjoint open connected sets.






            share|cite|improve this answer











            $endgroup$





















              1












              $begingroup$

              The balls with radii $frac{1}{n}$ and center at a rational number form a basis for the euclidean topology. This family is countable since $mathbb N times mathbb Q equiv mathbb N$ and we have a countable basis $(B_lambda)_{ , lambda in mathbb N times mathbb Q}$ of open intervals for $mathbb R$.



              Let $U$ be a nonempty open set in $mathbb R$; we can express it as countable union of open balls from $(B_lambda)$.
              Also,
              $tag 1 text{ }$
              $quad$ If two open intervals have a nonempty intersection, then their union is also an open interval.



              So if $U$ is a finite union of the $B_lambda$, it is an easy matter to combine the $B_lambda$, if necessary, and writing $U$ as a disjoint union of a finite number of open intervals. So assume, WLOG, that



              $tag 2 U = bigcup_{, n in mathbb N ,} B_n$.



              We define a relation on our (new) index set $mathbb N$ with $msim n$ if $B_m cap B_n ne emptyset$ or there is is a finite 'nonempty intersection $Btext{-}$chain' connecting $B_m$ with $B_n$. It is easy to see that this partitions $mathbb N$ and that taking the corresponding unions of the $B_n$ over an index $lambda text{-}$ block gives a partition of $U$. Also, using (1), we can show that we have also expressed $U$ as a countable union of disjoint open intervals.






              share|cite|improve this answer











              $endgroup$





















                1












                $begingroup$

                More of a question than answer. I am a chemist turned pharmacist, who wishes to have studied mathematics. I am trying to work through Rudin's Principles of Mathematical Analysis. I envy you all who are involved in math for a career.



                Can someone give me feedback on my attempt at a proof in $mathbb R? Completely novice and not at all pretty, but is it sound?



                segment := open interval in R.



                Lemma: disjoint segments in R are separated (proof not shown).



                It follows from the lemma that an open connected subset of R cannot be the union of disjoint segments.



                Let E be an open subset of R. Since R is separable by Rudin prob 2.22, there exists a subset, D, of R that is countable and dense in R. Assume E is connected, which includes E=R, then E is the union of an at most countable collection of open segments, containing only E.



                Suppose E is separated. Then E is the union of a collection of disjoint segments, including the possibility of segments unbounded above or below.



                If the collection is finite, then it is at most countable.



                Assume the collection of segments is infinite. Because D is dense in R and E is contained in R, every open subset of E contains a point of D. Then each of the infinitely many disjoint segments contains a unique point of D. Since D is countable, a one-to-one correspondence between a unique point of D and the segment is established. This implies that there are countably many disjoint segments in the collection.



                Therefore E is the union of finitely many or countably many
                , hence at most countably many, disjoint segments.






                share|cite|improve this answer











                $endgroup$









                • 1




                  $begingroup$
                  More of a question than answer. I suppose this would be better posted as a question than as an answer, then. Suppose E is separated. Then E is the union of a collection of disjoint segments. It's not obvious (to me, at least) where this follows from in the given context.
                  $endgroup$
                  – dxiv
                  Oct 9 '16 at 5:43








                • 1




                  $begingroup$
                  I know. Just learning how things work on this site. Which I think is quite amazing.
                  $endgroup$
                  – RJM
                  Oct 9 '16 at 5:48










                • $begingroup$
                  @dxiv Since E separated (or not connected), would it have to at least be the union of 2 separated sets. Since it is open, they have to be open subsets, which implies open interval in R1. Thank you!
                  $endgroup$
                  – RJM
                  Oct 9 '16 at 5:53












                • $begingroup$
                  @dxiv i will not keep bugging you. Any tips on how to become mathematician-like on one's own in essentially matgematica isolation?
                  $endgroup$
                  – RJM
                  Oct 9 '16 at 6:00










                • $begingroup$
                  IMHO that latter implies open interval in R1 is a part that needs to be proved in this context. See for example Intervals are connected and the only connected sets in R. Anyway, as I said, yours appears to be better suited as a question than as an answer. P.S. Keep plugging away until it all starts making sense ;-)
                  $endgroup$
                  – dxiv
                  Oct 9 '16 at 6:00














                Your Answer








                StackExchange.ready(function() {
                var channelOptions = {
                tags: "".split(" "),
                id: "69"
                };
                initTagRenderer("".split(" "), "".split(" "), channelOptions);

                StackExchange.using("externalEditor", function() {
                // Have to fire editor after snippets, if snippets enabled
                if (StackExchange.settings.snippets.snippetsEnabled) {
                StackExchange.using("snippets", function() {
                createEditor();
                });
                }
                else {
                createEditor();
                }
                });

                function createEditor() {
                StackExchange.prepareEditor({
                heartbeatType: 'answer',
                autoActivateHeartbeat: false,
                convertImagesToLinks: true,
                noModals: true,
                showLowRepImageUploadWarning: true,
                reputationToPostImages: 10,
                bindNavPrevention: true,
                postfix: "",
                imageUploader: {
                brandingHtml: "Powered by u003ca class="icon-imgur-white" href="https://imgur.com/"u003eu003c/au003e",
                contentPolicyHtml: "User contributions licensed under u003ca href="https://creativecommons.org/licenses/by-sa/3.0/"u003ecc by-sa 3.0 with attribution requiredu003c/au003e u003ca href="https://stackoverflow.com/legal/content-policy"u003e(content policy)u003c/au003e",
                allowUrls: true
                },
                noCode: true, onDemand: true,
                discardSelector: ".discard-answer"
                ,immediatelyShowMarkdownHelp:true
                });


                }
                });














                draft saved

                draft discarded


















                StackExchange.ready(
                function () {
                StackExchange.openid.initPostLogin('.new-post-login', 'https%3a%2f%2fmath.stackexchange.com%2fquestions%2f318299%2fany-open-subset-of-bbb-r-is-a-at-most-countable-union-of-disjoint-open-interv%23new-answer', 'question_page');
                }
                );

                Post as a guest















                Required, but never shown

























                16 Answers
                16






                active

                oldest

                votes








                16 Answers
                16






                active

                oldest

                votes









                active

                oldest

                votes






                active

                oldest

                votes









                100





                +100







                $begingroup$

                Here’s one to get things started.



                Let $U$ be a non-empty open subset of $Bbb R$. For $x,yin U$ define $xsim y$ iff $big[min{x,y},max{x,y}big]subseteq U$. It’s easily checked that $sim$ is an equivalence relation on $U$ whose equivalence classes are pairwise disjoint open intervals in $Bbb R$. (The term interval here includes unbounded intervals, i.e., rays.) Let $mathscr{I}$ be the set of $sim$-classes. Clearly $U=bigcup_{I in mathscr{I}} I$. For each $Iinmathscr{I}$ choose a rational $q_Iin I$; the map $mathscr{I}toBbb Q:Imapsto q_I$ is injective, so $mathscr{I}$ is countable.



                A variant of the same basic idea is to let $mathscr{I}$ be the set of open intervals that are subsets of $U$. For $I,Jinmathscr{I}$ define $Isim J$ iff there are $I_0=I,I_1,dots,I_n=Jinmathscr{I}$ such that $I_kcap I_{k+1}nevarnothing$ for $k=0,dots,n-1$. Then $sim$ is an equivalence relation on $mathscr{I}$. For $Iinmathscr{I}$ let $[I]$ be the $sim$-class of $I$. Then $left{bigcup[I]:Iinmathscr{I}right}$ is a decomposition of $U$ into pairwise disjoint open intervals.



                Both of these arguments generalize to any LOTS (= Linearly Ordered Topological Space), i.e., any linearly ordered set $langle X,lerangle$ with the topology generated by the subbase of open rays $(leftarrow,x)$ and $(x,to)$: if $U$ is a non-empty open subset of $X$, then $U$ is the union of a family of pairwise disjoint open intervals. In general the family need not be countable, of course.






                share|cite|improve this answer











                $endgroup$









                • 10




                  $begingroup$
                  I like this answer LOTS. :-) And +1.
                  $endgroup$
                  – coffeemath
                  Mar 2 '13 at 1:30










                • $begingroup$
                  This answer is very clear. It depends on the axiom of choice though, are there any constructive variants of this argument?
                  $endgroup$
                  – Bunder
                  Mar 2 '13 at 8:28






                • 6




                  $begingroup$
                  @Bunder: No, it doesn’t depend on the axiom of choice. The only place where you might even think that it did is when I showed that there are only countably many intervals, but the rationals can be explicitly well-ordered, so no choice is necessary even there.
                  $endgroup$
                  – Brian M. Scott
                  Mar 2 '13 at 12:24










                • $begingroup$
                  Is it correct to write each equivalence class as $I_x = (inf{y in U: y sim x}, sup{y in U: y sim x})$?
                  $endgroup$
                  – David
                  Nov 4 '15 at 16:09








                • 1




                  $begingroup$
                  @IntegrateThis. Let $S$ be a $sim$-equivalence class. For any $xin S$ there exists $r>0$ such that $(-r+x,r+x)subset U$ so $forall yin (-r+x,r+x);(ysim x). $ So $(-r+x,r+x)subset {yin U:ysim x}=S.$
                  $endgroup$
                  – DanielWainfleet
                  Sep 24 '18 at 20:38


















                100





                +100







                $begingroup$

                Here’s one to get things started.



                Let $U$ be a non-empty open subset of $Bbb R$. For $x,yin U$ define $xsim y$ iff $big[min{x,y},max{x,y}big]subseteq U$. It’s easily checked that $sim$ is an equivalence relation on $U$ whose equivalence classes are pairwise disjoint open intervals in $Bbb R$. (The term interval here includes unbounded intervals, i.e., rays.) Let $mathscr{I}$ be the set of $sim$-classes. Clearly $U=bigcup_{I in mathscr{I}} I$. For each $Iinmathscr{I}$ choose a rational $q_Iin I$; the map $mathscr{I}toBbb Q:Imapsto q_I$ is injective, so $mathscr{I}$ is countable.



                A variant of the same basic idea is to let $mathscr{I}$ be the set of open intervals that are subsets of $U$. For $I,Jinmathscr{I}$ define $Isim J$ iff there are $I_0=I,I_1,dots,I_n=Jinmathscr{I}$ such that $I_kcap I_{k+1}nevarnothing$ for $k=0,dots,n-1$. Then $sim$ is an equivalence relation on $mathscr{I}$. For $Iinmathscr{I}$ let $[I]$ be the $sim$-class of $I$. Then $left{bigcup[I]:Iinmathscr{I}right}$ is a decomposition of $U$ into pairwise disjoint open intervals.



                Both of these arguments generalize to any LOTS (= Linearly Ordered Topological Space), i.e., any linearly ordered set $langle X,lerangle$ with the topology generated by the subbase of open rays $(leftarrow,x)$ and $(x,to)$: if $U$ is a non-empty open subset of $X$, then $U$ is the union of a family of pairwise disjoint open intervals. In general the family need not be countable, of course.






                share|cite|improve this answer











                $endgroup$









                • 10




                  $begingroup$
                  I like this answer LOTS. :-) And +1.
                  $endgroup$
                  – coffeemath
                  Mar 2 '13 at 1:30










                • $begingroup$
                  This answer is very clear. It depends on the axiom of choice though, are there any constructive variants of this argument?
                  $endgroup$
                  – Bunder
                  Mar 2 '13 at 8:28






                • 6




                  $begingroup$
                  @Bunder: No, it doesn’t depend on the axiom of choice. The only place where you might even think that it did is when I showed that there are only countably many intervals, but the rationals can be explicitly well-ordered, so no choice is necessary even there.
                  $endgroup$
                  – Brian M. Scott
                  Mar 2 '13 at 12:24










                • $begingroup$
                  Is it correct to write each equivalence class as $I_x = (inf{y in U: y sim x}, sup{y in U: y sim x})$?
                  $endgroup$
                  – David
                  Nov 4 '15 at 16:09








                • 1




                  $begingroup$
                  @IntegrateThis. Let $S$ be a $sim$-equivalence class. For any $xin S$ there exists $r>0$ such that $(-r+x,r+x)subset U$ so $forall yin (-r+x,r+x);(ysim x). $ So $(-r+x,r+x)subset {yin U:ysim x}=S.$
                  $endgroup$
                  – DanielWainfleet
                  Sep 24 '18 at 20:38
















                100





                +100







                100





                +100



                100




                +100



                $begingroup$

                Here’s one to get things started.



                Let $U$ be a non-empty open subset of $Bbb R$. For $x,yin U$ define $xsim y$ iff $big[min{x,y},max{x,y}big]subseteq U$. It’s easily checked that $sim$ is an equivalence relation on $U$ whose equivalence classes are pairwise disjoint open intervals in $Bbb R$. (The term interval here includes unbounded intervals, i.e., rays.) Let $mathscr{I}$ be the set of $sim$-classes. Clearly $U=bigcup_{I in mathscr{I}} I$. For each $Iinmathscr{I}$ choose a rational $q_Iin I$; the map $mathscr{I}toBbb Q:Imapsto q_I$ is injective, so $mathscr{I}$ is countable.



                A variant of the same basic idea is to let $mathscr{I}$ be the set of open intervals that are subsets of $U$. For $I,Jinmathscr{I}$ define $Isim J$ iff there are $I_0=I,I_1,dots,I_n=Jinmathscr{I}$ such that $I_kcap I_{k+1}nevarnothing$ for $k=0,dots,n-1$. Then $sim$ is an equivalence relation on $mathscr{I}$. For $Iinmathscr{I}$ let $[I]$ be the $sim$-class of $I$. Then $left{bigcup[I]:Iinmathscr{I}right}$ is a decomposition of $U$ into pairwise disjoint open intervals.



                Both of these arguments generalize to any LOTS (= Linearly Ordered Topological Space), i.e., any linearly ordered set $langle X,lerangle$ with the topology generated by the subbase of open rays $(leftarrow,x)$ and $(x,to)$: if $U$ is a non-empty open subset of $X$, then $U$ is the union of a family of pairwise disjoint open intervals. In general the family need not be countable, of course.






                share|cite|improve this answer











                $endgroup$



                Here’s one to get things started.



                Let $U$ be a non-empty open subset of $Bbb R$. For $x,yin U$ define $xsim y$ iff $big[min{x,y},max{x,y}big]subseteq U$. It’s easily checked that $sim$ is an equivalence relation on $U$ whose equivalence classes are pairwise disjoint open intervals in $Bbb R$. (The term interval here includes unbounded intervals, i.e., rays.) Let $mathscr{I}$ be the set of $sim$-classes. Clearly $U=bigcup_{I in mathscr{I}} I$. For each $Iinmathscr{I}$ choose a rational $q_Iin I$; the map $mathscr{I}toBbb Q:Imapsto q_I$ is injective, so $mathscr{I}$ is countable.



                A variant of the same basic idea is to let $mathscr{I}$ be the set of open intervals that are subsets of $U$. For $I,Jinmathscr{I}$ define $Isim J$ iff there are $I_0=I,I_1,dots,I_n=Jinmathscr{I}$ such that $I_kcap I_{k+1}nevarnothing$ for $k=0,dots,n-1$. Then $sim$ is an equivalence relation on $mathscr{I}$. For $Iinmathscr{I}$ let $[I]$ be the $sim$-class of $I$. Then $left{bigcup[I]:Iinmathscr{I}right}$ is a decomposition of $U$ into pairwise disjoint open intervals.



                Both of these arguments generalize to any LOTS (= Linearly Ordered Topological Space), i.e., any linearly ordered set $langle X,lerangle$ with the topology generated by the subbase of open rays $(leftarrow,x)$ and $(x,to)$: if $U$ is a non-empty open subset of $X$, then $U$ is the union of a family of pairwise disjoint open intervals. In general the family need not be countable, of course.







                share|cite|improve this answer














                share|cite|improve this answer



                share|cite|improve this answer








                edited Jun 26 '15 at 14:00


























                community wiki





                4 revs, 3 users 80%
                Brian M. Scott









                • 10




                  $begingroup$
                  I like this answer LOTS. :-) And +1.
                  $endgroup$
                  – coffeemath
                  Mar 2 '13 at 1:30










                • $begingroup$
                  This answer is very clear. It depends on the axiom of choice though, are there any constructive variants of this argument?
                  $endgroup$
                  – Bunder
                  Mar 2 '13 at 8:28






                • 6




                  $begingroup$
                  @Bunder: No, it doesn’t depend on the axiom of choice. The only place where you might even think that it did is when I showed that there are only countably many intervals, but the rationals can be explicitly well-ordered, so no choice is necessary even there.
                  $endgroup$
                  – Brian M. Scott
                  Mar 2 '13 at 12:24










                • $begingroup$
                  Is it correct to write each equivalence class as $I_x = (inf{y in U: y sim x}, sup{y in U: y sim x})$?
                  $endgroup$
                  – David
                  Nov 4 '15 at 16:09








                • 1




                  $begingroup$
                  @IntegrateThis. Let $S$ be a $sim$-equivalence class. For any $xin S$ there exists $r>0$ such that $(-r+x,r+x)subset U$ so $forall yin (-r+x,r+x);(ysim x). $ So $(-r+x,r+x)subset {yin U:ysim x}=S.$
                  $endgroup$
                  – DanielWainfleet
                  Sep 24 '18 at 20:38
















                • 10




                  $begingroup$
                  I like this answer LOTS. :-) And +1.
                  $endgroup$
                  – coffeemath
                  Mar 2 '13 at 1:30










                • $begingroup$
                  This answer is very clear. It depends on the axiom of choice though, are there any constructive variants of this argument?
                  $endgroup$
                  – Bunder
                  Mar 2 '13 at 8:28






                • 6




                  $begingroup$
                  @Bunder: No, it doesn’t depend on the axiom of choice. The only place where you might even think that it did is when I showed that there are only countably many intervals, but the rationals can be explicitly well-ordered, so no choice is necessary even there.
                  $endgroup$
                  – Brian M. Scott
                  Mar 2 '13 at 12:24










                • $begingroup$
                  Is it correct to write each equivalence class as $I_x = (inf{y in U: y sim x}, sup{y in U: y sim x})$?
                  $endgroup$
                  – David
                  Nov 4 '15 at 16:09








                • 1




                  $begingroup$
                  @IntegrateThis. Let $S$ be a $sim$-equivalence class. For any $xin S$ there exists $r>0$ such that $(-r+x,r+x)subset U$ so $forall yin (-r+x,r+x);(ysim x). $ So $(-r+x,r+x)subset {yin U:ysim x}=S.$
                  $endgroup$
                  – DanielWainfleet
                  Sep 24 '18 at 20:38










                10




                10




                $begingroup$
                I like this answer LOTS. :-) And +1.
                $endgroup$
                – coffeemath
                Mar 2 '13 at 1:30




                $begingroup$
                I like this answer LOTS. :-) And +1.
                $endgroup$
                – coffeemath
                Mar 2 '13 at 1:30












                $begingroup$
                This answer is very clear. It depends on the axiom of choice though, are there any constructive variants of this argument?
                $endgroup$
                – Bunder
                Mar 2 '13 at 8:28




                $begingroup$
                This answer is very clear. It depends on the axiom of choice though, are there any constructive variants of this argument?
                $endgroup$
                – Bunder
                Mar 2 '13 at 8:28




                6




                6




                $begingroup$
                @Bunder: No, it doesn’t depend on the axiom of choice. The only place where you might even think that it did is when I showed that there are only countably many intervals, but the rationals can be explicitly well-ordered, so no choice is necessary even there.
                $endgroup$
                – Brian M. Scott
                Mar 2 '13 at 12:24




                $begingroup$
                @Bunder: No, it doesn’t depend on the axiom of choice. The only place where you might even think that it did is when I showed that there are only countably many intervals, but the rationals can be explicitly well-ordered, so no choice is necessary even there.
                $endgroup$
                – Brian M. Scott
                Mar 2 '13 at 12:24












                $begingroup$
                Is it correct to write each equivalence class as $I_x = (inf{y in U: y sim x}, sup{y in U: y sim x})$?
                $endgroup$
                – David
                Nov 4 '15 at 16:09






                $begingroup$
                Is it correct to write each equivalence class as $I_x = (inf{y in U: y sim x}, sup{y in U: y sim x})$?
                $endgroup$
                – David
                Nov 4 '15 at 16:09






                1




                1




                $begingroup$
                @IntegrateThis. Let $S$ be a $sim$-equivalence class. For any $xin S$ there exists $r>0$ such that $(-r+x,r+x)subset U$ so $forall yin (-r+x,r+x);(ysim x). $ So $(-r+x,r+x)subset {yin U:ysim x}=S.$
                $endgroup$
                – DanielWainfleet
                Sep 24 '18 at 20:38






                $begingroup$
                @IntegrateThis. Let $S$ be a $sim$-equivalence class. For any $xin S$ there exists $r>0$ such that $(-r+x,r+x)subset U$ so $forall yin (-r+x,r+x);(ysim x). $ So $(-r+x,r+x)subset {yin U:ysim x}=S.$
                $endgroup$
                – DanielWainfleet
                Sep 24 '18 at 20:38













                30












                $begingroup$

                These answers all seem to be variations on one another, but I've found each one so far to be at least a little cryptic. Here's my version/adaptation.



                Let $U subseteq R$ be open and let $x in U$. Either $x$ is rational or irrational. If $x$ is rational, define
                begin{align}I_x = bigcuplimits_{substack{Itext{ an open interval} \ x~in~I~subseteq~U}} I,end{align}
                which, as a union of non-disjoint open intervals (each $I$ contains $x$), is an open interval subset to $U$. If $x$ is irrational, by openness of $U$ there is $varepsilon > 0$ such that $(x - varepsilon, x + varepsilon) subseteq U$, and there exists rational $y in (x - varepsilon, x + varepsilon) subseteq I_y$ (by the definition of $I_y$). Hence $x in I_y$. So any $x in U$ is in $I_q$ for some $q in U cap mathbb{Q}$, and so
                begin{align}U subseteq bigcuplimits_{q~in~U cap~mathbb{Q}} I_q.end{align}
                But $I_q subseteq U$ for each $q in U cap mathbb{Q}$; thus
                begin{align}U = bigcuplimits_{q~in~U cap~mathbb{Q}} I_q, end{align}
                which is a countable union of open intervals.






                share|cite|improve this answer











                $endgroup$













                • $begingroup$
                  He wanted a countable union of disjoint open intervals. If $U$ is simply an open interval, then $U=I_q=I_{tilde{q}}$ for all $q,tilde{q} in mathcal{Q}cap U$.
                  $endgroup$
                  – José Siqueira
                  Oct 15 '13 at 16:15








                • 2




                  $begingroup$
                  That it is a disjoint union follows from the definition of $I_q$: if $x in I_q cap I_p$ then $I_q cup I_p subseteq I_q$ and $I_p$; hence if $I_q neq I_p$ then $I_q cap I_p = emptyset$. Strictly speaking one should throw away all repeated $I_q$s, and one can definitely do this without destroying the countability of the union.
                  $endgroup$
                  – Stromael
                  Oct 21 '13 at 11:14
















                30












                $begingroup$

                These answers all seem to be variations on one another, but I've found each one so far to be at least a little cryptic. Here's my version/adaptation.



                Let $U subseteq R$ be open and let $x in U$. Either $x$ is rational or irrational. If $x$ is rational, define
                begin{align}I_x = bigcuplimits_{substack{Itext{ an open interval} \ x~in~I~subseteq~U}} I,end{align}
                which, as a union of non-disjoint open intervals (each $I$ contains $x$), is an open interval subset to $U$. If $x$ is irrational, by openness of $U$ there is $varepsilon > 0$ such that $(x - varepsilon, x + varepsilon) subseteq U$, and there exists rational $y in (x - varepsilon, x + varepsilon) subseteq I_y$ (by the definition of $I_y$). Hence $x in I_y$. So any $x in U$ is in $I_q$ for some $q in U cap mathbb{Q}$, and so
                begin{align}U subseteq bigcuplimits_{q~in~U cap~mathbb{Q}} I_q.end{align}
                But $I_q subseteq U$ for each $q in U cap mathbb{Q}$; thus
                begin{align}U = bigcuplimits_{q~in~U cap~mathbb{Q}} I_q, end{align}
                which is a countable union of open intervals.






                share|cite|improve this answer











                $endgroup$













                • $begingroup$
                  He wanted a countable union of disjoint open intervals. If $U$ is simply an open interval, then $U=I_q=I_{tilde{q}}$ for all $q,tilde{q} in mathcal{Q}cap U$.
                  $endgroup$
                  – José Siqueira
                  Oct 15 '13 at 16:15








                • 2




                  $begingroup$
                  That it is a disjoint union follows from the definition of $I_q$: if $x in I_q cap I_p$ then $I_q cup I_p subseteq I_q$ and $I_p$; hence if $I_q neq I_p$ then $I_q cap I_p = emptyset$. Strictly speaking one should throw away all repeated $I_q$s, and one can definitely do this without destroying the countability of the union.
                  $endgroup$
                  – Stromael
                  Oct 21 '13 at 11:14














                30












                30








                30





                $begingroup$

                These answers all seem to be variations on one another, but I've found each one so far to be at least a little cryptic. Here's my version/adaptation.



                Let $U subseteq R$ be open and let $x in U$. Either $x$ is rational or irrational. If $x$ is rational, define
                begin{align}I_x = bigcuplimits_{substack{Itext{ an open interval} \ x~in~I~subseteq~U}} I,end{align}
                which, as a union of non-disjoint open intervals (each $I$ contains $x$), is an open interval subset to $U$. If $x$ is irrational, by openness of $U$ there is $varepsilon > 0$ such that $(x - varepsilon, x + varepsilon) subseteq U$, and there exists rational $y in (x - varepsilon, x + varepsilon) subseteq I_y$ (by the definition of $I_y$). Hence $x in I_y$. So any $x in U$ is in $I_q$ for some $q in U cap mathbb{Q}$, and so
                begin{align}U subseteq bigcuplimits_{q~in~U cap~mathbb{Q}} I_q.end{align}
                But $I_q subseteq U$ for each $q in U cap mathbb{Q}$; thus
                begin{align}U = bigcuplimits_{q~in~U cap~mathbb{Q}} I_q, end{align}
                which is a countable union of open intervals.






                share|cite|improve this answer











                $endgroup$



                These answers all seem to be variations on one another, but I've found each one so far to be at least a little cryptic. Here's my version/adaptation.



                Let $U subseteq R$ be open and let $x in U$. Either $x$ is rational or irrational. If $x$ is rational, define
                begin{align}I_x = bigcuplimits_{substack{Itext{ an open interval} \ x~in~I~subseteq~U}} I,end{align}
                which, as a union of non-disjoint open intervals (each $I$ contains $x$), is an open interval subset to $U$. If $x$ is irrational, by openness of $U$ there is $varepsilon > 0$ such that $(x - varepsilon, x + varepsilon) subseteq U$, and there exists rational $y in (x - varepsilon, x + varepsilon) subseteq I_y$ (by the definition of $I_y$). Hence $x in I_y$. So any $x in U$ is in $I_q$ for some $q in U cap mathbb{Q}$, and so
                begin{align}U subseteq bigcuplimits_{q~in~U cap~mathbb{Q}} I_q.end{align}
                But $I_q subseteq U$ for each $q in U cap mathbb{Q}$; thus
                begin{align}U = bigcuplimits_{q~in~U cap~mathbb{Q}} I_q, end{align}
                which is a countable union of open intervals.







                share|cite|improve this answer














                share|cite|improve this answer



                share|cite|improve this answer








                answered May 19 '13 at 11:45


























                community wiki





                Stromael













                • $begingroup$
                  He wanted a countable union of disjoint open intervals. If $U$ is simply an open interval, then $U=I_q=I_{tilde{q}}$ for all $q,tilde{q} in mathcal{Q}cap U$.
                  $endgroup$
                  – José Siqueira
                  Oct 15 '13 at 16:15








                • 2




                  $begingroup$
                  That it is a disjoint union follows from the definition of $I_q$: if $x in I_q cap I_p$ then $I_q cup I_p subseteq I_q$ and $I_p$; hence if $I_q neq I_p$ then $I_q cap I_p = emptyset$. Strictly speaking one should throw away all repeated $I_q$s, and one can definitely do this without destroying the countability of the union.
                  $endgroup$
                  – Stromael
                  Oct 21 '13 at 11:14


















                • $begingroup$
                  He wanted a countable union of disjoint open intervals. If $U$ is simply an open interval, then $U=I_q=I_{tilde{q}}$ for all $q,tilde{q} in mathcal{Q}cap U$.
                  $endgroup$
                  – José Siqueira
                  Oct 15 '13 at 16:15








                • 2




                  $begingroup$
                  That it is a disjoint union follows from the definition of $I_q$: if $x in I_q cap I_p$ then $I_q cup I_p subseteq I_q$ and $I_p$; hence if $I_q neq I_p$ then $I_q cap I_p = emptyset$. Strictly speaking one should throw away all repeated $I_q$s, and one can definitely do this without destroying the countability of the union.
                  $endgroup$
                  – Stromael
                  Oct 21 '13 at 11:14
















                $begingroup$
                He wanted a countable union of disjoint open intervals. If $U$ is simply an open interval, then $U=I_q=I_{tilde{q}}$ for all $q,tilde{q} in mathcal{Q}cap U$.
                $endgroup$
                – José Siqueira
                Oct 15 '13 at 16:15






                $begingroup$
                He wanted a countable union of disjoint open intervals. If $U$ is simply an open interval, then $U=I_q=I_{tilde{q}}$ for all $q,tilde{q} in mathcal{Q}cap U$.
                $endgroup$
                – José Siqueira
                Oct 15 '13 at 16:15






                2




                2




                $begingroup$
                That it is a disjoint union follows from the definition of $I_q$: if $x in I_q cap I_p$ then $I_q cup I_p subseteq I_q$ and $I_p$; hence if $I_q neq I_p$ then $I_q cap I_p = emptyset$. Strictly speaking one should throw away all repeated $I_q$s, and one can definitely do this without destroying the countability of the union.
                $endgroup$
                – Stromael
                Oct 21 '13 at 11:14




                $begingroup$
                That it is a disjoint union follows from the definition of $I_q$: if $x in I_q cap I_p$ then $I_q cup I_p subseteq I_q$ and $I_p$; hence if $I_q neq I_p$ then $I_q cap I_p = emptyset$. Strictly speaking one should throw away all repeated $I_q$s, and one can definitely do this without destroying the countability of the union.
                $endgroup$
                – Stromael
                Oct 21 '13 at 11:14











                15












                $begingroup$

                In a locally connected space $X$, all connected components of open sets are open. This is in fact equivalent to being locally connected.



                Proof: (one direction) let $O$ be an open subset of a locally connected space $X$. Let $C$ be a component of $O$ (as a (sub)space in its own right). Let $x in C$. Then let $U_x$ be a connected neighbourhood of $x$ in $X$ such that $U_x subset O$, which can be done as $O$ is open and the connected neighbourhoods form a local base. Then $U_x,C subset O$ are both connected and intersect (in $x$) so their union $U_x cup C subset O$ is a connected subset of $O$ containing $x$, so by maximality of components $U_x cup C subset C$. But then $U_x$ witnesses that $x$ is an interior point of $C$, and this shows all points of $C$ are interior points, hence $C$ is open (in either $X$ or $O$, that's equivalent).



                Now $mathbb{R}$ is locally connected (open intervals form a local base of connected sets) and so every open set if a disjoint union of its components, which are open connected subsets of $mathbb{R}$, hence are open intervals (potentially of infinite "length", i.e. segments). That there are countably many of them at most, follows from the already given "rational in every interval" argument.






                share|cite|improve this answer











                $endgroup$













                • $begingroup$
                  "connected => interval" also requires a proof (usually using the intermediate step "path-connected").
                  $endgroup$
                  – Martin Brandenburg
                  Mar 4 '13 at 19:29






                • 1




                  $begingroup$
                  Path connected is not needed. It would be circular, as one needs connectedness of intervals to see that path-connected implies connected... If a set is not order convex (so x < z < y, x,y in A but z not in A), then we have an immediate disconnection. That intervals are connected follows from completeness of the order, and is a standard fact for ordered spaces.
                  $endgroup$
                  – Henno Brandsma
                  Mar 4 '13 at 20:22
















                15












                $begingroup$

                In a locally connected space $X$, all connected components of open sets are open. This is in fact equivalent to being locally connected.



                Proof: (one direction) let $O$ be an open subset of a locally connected space $X$. Let $C$ be a component of $O$ (as a (sub)space in its own right). Let $x in C$. Then let $U_x$ be a connected neighbourhood of $x$ in $X$ such that $U_x subset O$, which can be done as $O$ is open and the connected neighbourhoods form a local base. Then $U_x,C subset O$ are both connected and intersect (in $x$) so their union $U_x cup C subset O$ is a connected subset of $O$ containing $x$, so by maximality of components $U_x cup C subset C$. But then $U_x$ witnesses that $x$ is an interior point of $C$, and this shows all points of $C$ are interior points, hence $C$ is open (in either $X$ or $O$, that's equivalent).



                Now $mathbb{R}$ is locally connected (open intervals form a local base of connected sets) and so every open set if a disjoint union of its components, which are open connected subsets of $mathbb{R}$, hence are open intervals (potentially of infinite "length", i.e. segments). That there are countably many of them at most, follows from the already given "rational in every interval" argument.






                share|cite|improve this answer











                $endgroup$













                • $begingroup$
                  "connected => interval" also requires a proof (usually using the intermediate step "path-connected").
                  $endgroup$
                  – Martin Brandenburg
                  Mar 4 '13 at 19:29






                • 1




                  $begingroup$
                  Path connected is not needed. It would be circular, as one needs connectedness of intervals to see that path-connected implies connected... If a set is not order convex (so x < z < y, x,y in A but z not in A), then we have an immediate disconnection. That intervals are connected follows from completeness of the order, and is a standard fact for ordered spaces.
                  $endgroup$
                  – Henno Brandsma
                  Mar 4 '13 at 20:22














                15












                15








                15





                $begingroup$

                In a locally connected space $X$, all connected components of open sets are open. This is in fact equivalent to being locally connected.



                Proof: (one direction) let $O$ be an open subset of a locally connected space $X$. Let $C$ be a component of $O$ (as a (sub)space in its own right). Let $x in C$. Then let $U_x$ be a connected neighbourhood of $x$ in $X$ such that $U_x subset O$, which can be done as $O$ is open and the connected neighbourhoods form a local base. Then $U_x,C subset O$ are both connected and intersect (in $x$) so their union $U_x cup C subset O$ is a connected subset of $O$ containing $x$, so by maximality of components $U_x cup C subset C$. But then $U_x$ witnesses that $x$ is an interior point of $C$, and this shows all points of $C$ are interior points, hence $C$ is open (in either $X$ or $O$, that's equivalent).



                Now $mathbb{R}$ is locally connected (open intervals form a local base of connected sets) and so every open set if a disjoint union of its components, which are open connected subsets of $mathbb{R}$, hence are open intervals (potentially of infinite "length", i.e. segments). That there are countably many of them at most, follows from the already given "rational in every interval" argument.






                share|cite|improve this answer











                $endgroup$



                In a locally connected space $X$, all connected components of open sets are open. This is in fact equivalent to being locally connected.



                Proof: (one direction) let $O$ be an open subset of a locally connected space $X$. Let $C$ be a component of $O$ (as a (sub)space in its own right). Let $x in C$. Then let $U_x$ be a connected neighbourhood of $x$ in $X$ such that $U_x subset O$, which can be done as $O$ is open and the connected neighbourhoods form a local base. Then $U_x,C subset O$ are both connected and intersect (in $x$) so their union $U_x cup C subset O$ is a connected subset of $O$ containing $x$, so by maximality of components $U_x cup C subset C$. But then $U_x$ witnesses that $x$ is an interior point of $C$, and this shows all points of $C$ are interior points, hence $C$ is open (in either $X$ or $O$, that's equivalent).



                Now $mathbb{R}$ is locally connected (open intervals form a local base of connected sets) and so every open set if a disjoint union of its components, which are open connected subsets of $mathbb{R}$, hence are open intervals (potentially of infinite "length", i.e. segments). That there are countably many of them at most, follows from the already given "rational in every interval" argument.







                share|cite|improve this answer














                share|cite|improve this answer



                share|cite|improve this answer








                answered Mar 2 '13 at 7:06


























                community wiki





                Henno Brandsma













                • $begingroup$
                  "connected => interval" also requires a proof (usually using the intermediate step "path-connected").
                  $endgroup$
                  – Martin Brandenburg
                  Mar 4 '13 at 19:29






                • 1




                  $begingroup$
                  Path connected is not needed. It would be circular, as one needs connectedness of intervals to see that path-connected implies connected... If a set is not order convex (so x < z < y, x,y in A but z not in A), then we have an immediate disconnection. That intervals are connected follows from completeness of the order, and is a standard fact for ordered spaces.
                  $endgroup$
                  – Henno Brandsma
                  Mar 4 '13 at 20:22


















                • $begingroup$
                  "connected => interval" also requires a proof (usually using the intermediate step "path-connected").
                  $endgroup$
                  – Martin Brandenburg
                  Mar 4 '13 at 19:29






                • 1




                  $begingroup$
                  Path connected is not needed. It would be circular, as one needs connectedness of intervals to see that path-connected implies connected... If a set is not order convex (so x < z < y, x,y in A but z not in A), then we have an immediate disconnection. That intervals are connected follows from completeness of the order, and is a standard fact for ordered spaces.
                  $endgroup$
                  – Henno Brandsma
                  Mar 4 '13 at 20:22
















                $begingroup$
                "connected => interval" also requires a proof (usually using the intermediate step "path-connected").
                $endgroup$
                – Martin Brandenburg
                Mar 4 '13 at 19:29




                $begingroup$
                "connected => interval" also requires a proof (usually using the intermediate step "path-connected").
                $endgroup$
                – Martin Brandenburg
                Mar 4 '13 at 19:29




                1




                1




                $begingroup$
                Path connected is not needed. It would be circular, as one needs connectedness of intervals to see that path-connected implies connected... If a set is not order convex (so x < z < y, x,y in A but z not in A), then we have an immediate disconnection. That intervals are connected follows from completeness of the order, and is a standard fact for ordered spaces.
                $endgroup$
                – Henno Brandsma
                Mar 4 '13 at 20:22




                $begingroup$
                Path connected is not needed. It would be circular, as one needs connectedness of intervals to see that path-connected implies connected... If a set is not order convex (so x < z < y, x,y in A but z not in A), then we have an immediate disconnection. That intervals are connected follows from completeness of the order, and is a standard fact for ordered spaces.
                $endgroup$
                – Henno Brandsma
                Mar 4 '13 at 20:22











                11












                $begingroup$

                Let $Usubseteqmathbb R$ open. Is enough to write $U$ as a disjoint union of open intervals.

                For each $xin U$ we define $alpha_x=inf{alphainmathbb R:(alpha,x+epsilon)subseteq U, text{ for some }epsilon>0}$ and $beta_x=sup{betainmathbb R:(alpha_x,beta)subseteq U}$.



                Then $displaystyle U=bigcup_{xin U}(alpha_x,beta_x)$ where ${(alpha_x,beta_x):xin U}$ is a disjoint family of open intervals.






                share|cite|improve this answer











                $endgroup$









                • 1




                  $begingroup$
                  Are you sure the union is countable here?
                  $endgroup$
                  – user10444
                  Sep 11 '14 at 16:55






                • 1




                  $begingroup$
                  @user10444: It is countable yes! If you agree that ${(alpha_x,beta_x):xin U}$ is a disjoint family of open intervals then you can see it by choosing $r_xin mathbb Qcap (alpha_x,beta_x)$ for all $xin U$. Then ${r_x:xin U}$ is countable right? Note that the intervals ${(alpha_x,beta_x):xin U}$ are not distinct!
                  $endgroup$
                  – P..
                  Sep 11 '14 at 19:03








                • 1




                  $begingroup$
                  Recall, we can "choose" such an $r_{x}in(alpha_{x},beta_{x})$ such that $r_{x}inmathbb{Q}$ by the Axiom of Choice and by the density of $mathbb{Q}$ in $mathbb{R}$.
                  $endgroup$
                  – Procore
                  Aug 30 '17 at 20:29


















                11












                $begingroup$

                Let $Usubseteqmathbb R$ open. Is enough to write $U$ as a disjoint union of open intervals.

                For each $xin U$ we define $alpha_x=inf{alphainmathbb R:(alpha,x+epsilon)subseteq U, text{ for some }epsilon>0}$ and $beta_x=sup{betainmathbb R:(alpha_x,beta)subseteq U}$.



                Then $displaystyle U=bigcup_{xin U}(alpha_x,beta_x)$ where ${(alpha_x,beta_x):xin U}$ is a disjoint family of open intervals.






                share|cite|improve this answer











                $endgroup$









                • 1




                  $begingroup$
                  Are you sure the union is countable here?
                  $endgroup$
                  – user10444
                  Sep 11 '14 at 16:55






                • 1




                  $begingroup$
                  @user10444: It is countable yes! If you agree that ${(alpha_x,beta_x):xin U}$ is a disjoint family of open intervals then you can see it by choosing $r_xin mathbb Qcap (alpha_x,beta_x)$ for all $xin U$. Then ${r_x:xin U}$ is countable right? Note that the intervals ${(alpha_x,beta_x):xin U}$ are not distinct!
                  $endgroup$
                  – P..
                  Sep 11 '14 at 19:03








                • 1




                  $begingroup$
                  Recall, we can "choose" such an $r_{x}in(alpha_{x},beta_{x})$ such that $r_{x}inmathbb{Q}$ by the Axiom of Choice and by the density of $mathbb{Q}$ in $mathbb{R}$.
                  $endgroup$
                  – Procore
                  Aug 30 '17 at 20:29
















                11












                11








                11





                $begingroup$

                Let $Usubseteqmathbb R$ open. Is enough to write $U$ as a disjoint union of open intervals.

                For each $xin U$ we define $alpha_x=inf{alphainmathbb R:(alpha,x+epsilon)subseteq U, text{ for some }epsilon>0}$ and $beta_x=sup{betainmathbb R:(alpha_x,beta)subseteq U}$.



                Then $displaystyle U=bigcup_{xin U}(alpha_x,beta_x)$ where ${(alpha_x,beta_x):xin U}$ is a disjoint family of open intervals.






                share|cite|improve this answer











                $endgroup$



                Let $Usubseteqmathbb R$ open. Is enough to write $U$ as a disjoint union of open intervals.

                For each $xin U$ we define $alpha_x=inf{alphainmathbb R:(alpha,x+epsilon)subseteq U, text{ for some }epsilon>0}$ and $beta_x=sup{betainmathbb R:(alpha_x,beta)subseteq U}$.



                Then $displaystyle U=bigcup_{xin U}(alpha_x,beta_x)$ where ${(alpha_x,beta_x):xin U}$ is a disjoint family of open intervals.







                share|cite|improve this answer














                share|cite|improve this answer



                share|cite|improve this answer








                edited Mar 4 '13 at 19:53


























                community wiki





                3 revs
                P..









                • 1




                  $begingroup$
                  Are you sure the union is countable here?
                  $endgroup$
                  – user10444
                  Sep 11 '14 at 16:55






                • 1




                  $begingroup$
                  @user10444: It is countable yes! If you agree that ${(alpha_x,beta_x):xin U}$ is a disjoint family of open intervals then you can see it by choosing $r_xin mathbb Qcap (alpha_x,beta_x)$ for all $xin U$. Then ${r_x:xin U}$ is countable right? Note that the intervals ${(alpha_x,beta_x):xin U}$ are not distinct!
                  $endgroup$
                  – P..
                  Sep 11 '14 at 19:03








                • 1




                  $begingroup$
                  Recall, we can "choose" such an $r_{x}in(alpha_{x},beta_{x})$ such that $r_{x}inmathbb{Q}$ by the Axiom of Choice and by the density of $mathbb{Q}$ in $mathbb{R}$.
                  $endgroup$
                  – Procore
                  Aug 30 '17 at 20:29
















                • 1




                  $begingroup$
                  Are you sure the union is countable here?
                  $endgroup$
                  – user10444
                  Sep 11 '14 at 16:55






                • 1




                  $begingroup$
                  @user10444: It is countable yes! If you agree that ${(alpha_x,beta_x):xin U}$ is a disjoint family of open intervals then you can see it by choosing $r_xin mathbb Qcap (alpha_x,beta_x)$ for all $xin U$. Then ${r_x:xin U}$ is countable right? Note that the intervals ${(alpha_x,beta_x):xin U}$ are not distinct!
                  $endgroup$
                  – P..
                  Sep 11 '14 at 19:03








                • 1




                  $begingroup$
                  Recall, we can "choose" such an $r_{x}in(alpha_{x},beta_{x})$ such that $r_{x}inmathbb{Q}$ by the Axiom of Choice and by the density of $mathbb{Q}$ in $mathbb{R}$.
                  $endgroup$
                  – Procore
                  Aug 30 '17 at 20:29










                1




                1




                $begingroup$
                Are you sure the union is countable here?
                $endgroup$
                – user10444
                Sep 11 '14 at 16:55




                $begingroup$
                Are you sure the union is countable here?
                $endgroup$
                – user10444
                Sep 11 '14 at 16:55




                1




                1




                $begingroup$
                @user10444: It is countable yes! If you agree that ${(alpha_x,beta_x):xin U}$ is a disjoint family of open intervals then you can see it by choosing $r_xin mathbb Qcap (alpha_x,beta_x)$ for all $xin U$. Then ${r_x:xin U}$ is countable right? Note that the intervals ${(alpha_x,beta_x):xin U}$ are not distinct!
                $endgroup$
                – P..
                Sep 11 '14 at 19:03






                $begingroup$
                @user10444: It is countable yes! If you agree that ${(alpha_x,beta_x):xin U}$ is a disjoint family of open intervals then you can see it by choosing $r_xin mathbb Qcap (alpha_x,beta_x)$ for all $xin U$. Then ${r_x:xin U}$ is countable right? Note that the intervals ${(alpha_x,beta_x):xin U}$ are not distinct!
                $endgroup$
                – P..
                Sep 11 '14 at 19:03






                1




                1




                $begingroup$
                Recall, we can "choose" such an $r_{x}in(alpha_{x},beta_{x})$ such that $r_{x}inmathbb{Q}$ by the Axiom of Choice and by the density of $mathbb{Q}$ in $mathbb{R}$.
                $endgroup$
                – Procore
                Aug 30 '17 at 20:29






                $begingroup$
                Recall, we can "choose" such an $r_{x}in(alpha_{x},beta_{x})$ such that $r_{x}inmathbb{Q}$ by the Axiom of Choice and by the density of $mathbb{Q}$ in $mathbb{R}$.
                $endgroup$
                – Procore
                Aug 30 '17 at 20:29













                6












                $begingroup$

                A variant of the usual proof with the equivalence relation, which trades in the ease of constructing the intervals with the ease of proving countability (not that either is hard...):




                1. Define the same equivalence relation, but only on $mathbb Q cap U$:
                  $q_1 sim q_2$ iff $(q_1, q_2) subset U$ (or $(q_2, q_1) subset U$, whichever makes sense).

                2. From each equivalency class $C$, produce the open interval $(inf C, sup C) subset U$ (where $inf C$ is defined to be $-infty$ in case $C$ is not bounded from below, and $sup C = infty$ in case $C$ is not bounded from above).

                3. The amount of equivalence classes is clearly countable, since $mathbb Q cap U$ is countable.






                share|cite|improve this answer











                $endgroup$


















                  6












                  $begingroup$

                  A variant of the usual proof with the equivalence relation, which trades in the ease of constructing the intervals with the ease of proving countability (not that either is hard...):




                  1. Define the same equivalence relation, but only on $mathbb Q cap U$:
                    $q_1 sim q_2$ iff $(q_1, q_2) subset U$ (or $(q_2, q_1) subset U$, whichever makes sense).

                  2. From each equivalency class $C$, produce the open interval $(inf C, sup C) subset U$ (where $inf C$ is defined to be $-infty$ in case $C$ is not bounded from below, and $sup C = infty$ in case $C$ is not bounded from above).

                  3. The amount of equivalence classes is clearly countable, since $mathbb Q cap U$ is countable.






                  share|cite|improve this answer











                  $endgroup$
















                    6












                    6








                    6





                    $begingroup$

                    A variant of the usual proof with the equivalence relation, which trades in the ease of constructing the intervals with the ease of proving countability (not that either is hard...):




                    1. Define the same equivalence relation, but only on $mathbb Q cap U$:
                      $q_1 sim q_2$ iff $(q_1, q_2) subset U$ (or $(q_2, q_1) subset U$, whichever makes sense).

                    2. From each equivalency class $C$, produce the open interval $(inf C, sup C) subset U$ (where $inf C$ is defined to be $-infty$ in case $C$ is not bounded from below, and $sup C = infty$ in case $C$ is not bounded from above).

                    3. The amount of equivalence classes is clearly countable, since $mathbb Q cap U$ is countable.






                    share|cite|improve this answer











                    $endgroup$



                    A variant of the usual proof with the equivalence relation, which trades in the ease of constructing the intervals with the ease of proving countability (not that either is hard...):




                    1. Define the same equivalence relation, but only on $mathbb Q cap U$:
                      $q_1 sim q_2$ iff $(q_1, q_2) subset U$ (or $(q_2, q_1) subset U$, whichever makes sense).

                    2. From each equivalency class $C$, produce the open interval $(inf C, sup C) subset U$ (where $inf C$ is defined to be $-infty$ in case $C$ is not bounded from below, and $sup C = infty$ in case $C$ is not bounded from above).

                    3. The amount of equivalence classes is clearly countable, since $mathbb Q cap U$ is countable.







                    share|cite|improve this answer














                    share|cite|improve this answer



                    share|cite|improve this answer








                    answered Mar 2 '13 at 1:28


























                    community wiki





                    Yoni Rozenshein
























                        6












                        $begingroup$

                        Let $U$ be an open subset of $mathbb{R}$. Let $P$ be the poset consisting of collections $mathcal{A}$ of disjoint open intervals where we say $mathcal{A} le mathcal{A}'$ if each of the sets in $mathcal{A}$ is a subset of some open interval in $mathcal{A}'$. Every chain $C$ in this poset has an upper bound, namely $$mathcal{B} = left{ bigcupleft{J in bigcupbigcup C : I subseteq J right}: I in bigcupbigcup Cright}.$$
                        Therefore by Zorn's lemma the poset $P$ has a maximal element $mathcal{M}$. We claim that the union of the intervals in $mathcal{M}$ is all of $U$. Suppose toward a contradiction that there is a real $x in U$ that is not contained in any of the intervals in $mathcal{M}$. Because $U$ is open we can take an open interval $I$ with $x in I subseteq U$.
                        Then the set
                        $$mathcal{M}' = {J in mathcal{M} : J cap I = emptyset} cup left{I cup bigcup {J in mathcal{M} : J cap I ne emptyset}right}$$
                        is a collection of disjoint open intervals and
                        is above $mathcal{M}$ in the poset $P$, contradicting the maximality of $mathcal{M}$. It remains to observe that $mathcal{M}$ is countable, which follows from the fact that its elements contain distinct rational numbers.



                        Note that the only way in which anything about order (or connectedness) is used is to see that $I cup bigcup {J in mathcal{M} : J cap I ne emptyset}$ is an interval.






                        share|cite|improve this answer











                        $endgroup$













                        • $begingroup$
                          And yes, before you ask, I know this proof is silly.
                          $endgroup$
                          – Trevor Wilson
                          Mar 8 '13 at 2:21






                        • 2




                          $begingroup$
                          I think this proof conceptually connects many things, including well-ordering, order theory and so on. So, I really benefited.
                          $endgroup$
                          – user64066
                          Oct 22 '13 at 20:19










                        • $begingroup$
                          @user64066 Thanks, I'm glad to hear it.
                          $endgroup$
                          – Trevor Wilson
                          Oct 22 '13 at 20:30
















                        6












                        $begingroup$

                        Let $U$ be an open subset of $mathbb{R}$. Let $P$ be the poset consisting of collections $mathcal{A}$ of disjoint open intervals where we say $mathcal{A} le mathcal{A}'$ if each of the sets in $mathcal{A}$ is a subset of some open interval in $mathcal{A}'$. Every chain $C$ in this poset has an upper bound, namely $$mathcal{B} = left{ bigcupleft{J in bigcupbigcup C : I subseteq J right}: I in bigcupbigcup Cright}.$$
                        Therefore by Zorn's lemma the poset $P$ has a maximal element $mathcal{M}$. We claim that the union of the intervals in $mathcal{M}$ is all of $U$. Suppose toward a contradiction that there is a real $x in U$ that is not contained in any of the intervals in $mathcal{M}$. Because $U$ is open we can take an open interval $I$ with $x in I subseteq U$.
                        Then the set
                        $$mathcal{M}' = {J in mathcal{M} : J cap I = emptyset} cup left{I cup bigcup {J in mathcal{M} : J cap I ne emptyset}right}$$
                        is a collection of disjoint open intervals and
                        is above $mathcal{M}$ in the poset $P$, contradicting the maximality of $mathcal{M}$. It remains to observe that $mathcal{M}$ is countable, which follows from the fact that its elements contain distinct rational numbers.



                        Note that the only way in which anything about order (or connectedness) is used is to see that $I cup bigcup {J in mathcal{M} : J cap I ne emptyset}$ is an interval.






                        share|cite|improve this answer











                        $endgroup$













                        • $begingroup$
                          And yes, before you ask, I know this proof is silly.
                          $endgroup$
                          – Trevor Wilson
                          Mar 8 '13 at 2:21






                        • 2




                          $begingroup$
                          I think this proof conceptually connects many things, including well-ordering, order theory and so on. So, I really benefited.
                          $endgroup$
                          – user64066
                          Oct 22 '13 at 20:19










                        • $begingroup$
                          @user64066 Thanks, I'm glad to hear it.
                          $endgroup$
                          – Trevor Wilson
                          Oct 22 '13 at 20:30














                        6












                        6








                        6





                        $begingroup$

                        Let $U$ be an open subset of $mathbb{R}$. Let $P$ be the poset consisting of collections $mathcal{A}$ of disjoint open intervals where we say $mathcal{A} le mathcal{A}'$ if each of the sets in $mathcal{A}$ is a subset of some open interval in $mathcal{A}'$. Every chain $C$ in this poset has an upper bound, namely $$mathcal{B} = left{ bigcupleft{J in bigcupbigcup C : I subseteq J right}: I in bigcupbigcup Cright}.$$
                        Therefore by Zorn's lemma the poset $P$ has a maximal element $mathcal{M}$. We claim that the union of the intervals in $mathcal{M}$ is all of $U$. Suppose toward a contradiction that there is a real $x in U$ that is not contained in any of the intervals in $mathcal{M}$. Because $U$ is open we can take an open interval $I$ with $x in I subseteq U$.
                        Then the set
                        $$mathcal{M}' = {J in mathcal{M} : J cap I = emptyset} cup left{I cup bigcup {J in mathcal{M} : J cap I ne emptyset}right}$$
                        is a collection of disjoint open intervals and
                        is above $mathcal{M}$ in the poset $P$, contradicting the maximality of $mathcal{M}$. It remains to observe that $mathcal{M}$ is countable, which follows from the fact that its elements contain distinct rational numbers.



                        Note that the only way in which anything about order (or connectedness) is used is to see that $I cup bigcup {J in mathcal{M} : J cap I ne emptyset}$ is an interval.






                        share|cite|improve this answer











                        $endgroup$



                        Let $U$ be an open subset of $mathbb{R}$. Let $P$ be the poset consisting of collections $mathcal{A}$ of disjoint open intervals where we say $mathcal{A} le mathcal{A}'$ if each of the sets in $mathcal{A}$ is a subset of some open interval in $mathcal{A}'$. Every chain $C$ in this poset has an upper bound, namely $$mathcal{B} = left{ bigcupleft{J in bigcupbigcup C : I subseteq J right}: I in bigcupbigcup Cright}.$$
                        Therefore by Zorn's lemma the poset $P$ has a maximal element $mathcal{M}$. We claim that the union of the intervals in $mathcal{M}$ is all of $U$. Suppose toward a contradiction that there is a real $x in U$ that is not contained in any of the intervals in $mathcal{M}$. Because $U$ is open we can take an open interval $I$ with $x in I subseteq U$.
                        Then the set
                        $$mathcal{M}' = {J in mathcal{M} : J cap I = emptyset} cup left{I cup bigcup {J in mathcal{M} : J cap I ne emptyset}right}$$
                        is a collection of disjoint open intervals and
                        is above $mathcal{M}$ in the poset $P$, contradicting the maximality of $mathcal{M}$. It remains to observe that $mathcal{M}$ is countable, which follows from the fact that its elements contain distinct rational numbers.



                        Note that the only way in which anything about order (or connectedness) is used is to see that $I cup bigcup {J in mathcal{M} : J cap I ne emptyset}$ is an interval.







                        share|cite|improve this answer














                        share|cite|improve this answer



                        share|cite|improve this answer








                        edited Mar 8 '13 at 2:17


























                        community wiki





                        2 revs
                        Trevor Wilson













                        • $begingroup$
                          And yes, before you ask, I know this proof is silly.
                          $endgroup$
                          – Trevor Wilson
                          Mar 8 '13 at 2:21






                        • 2




                          $begingroup$
                          I think this proof conceptually connects many things, including well-ordering, order theory and so on. So, I really benefited.
                          $endgroup$
                          – user64066
                          Oct 22 '13 at 20:19










                        • $begingroup$
                          @user64066 Thanks, I'm glad to hear it.
                          $endgroup$
                          – Trevor Wilson
                          Oct 22 '13 at 20:30


















                        • $begingroup$
                          And yes, before you ask, I know this proof is silly.
                          $endgroup$
                          – Trevor Wilson
                          Mar 8 '13 at 2:21






                        • 2




                          $begingroup$
                          I think this proof conceptually connects many things, including well-ordering, order theory and so on. So, I really benefited.
                          $endgroup$
                          – user64066
                          Oct 22 '13 at 20:19










                        • $begingroup$
                          @user64066 Thanks, I'm glad to hear it.
                          $endgroup$
                          – Trevor Wilson
                          Oct 22 '13 at 20:30
















                        $begingroup$
                        And yes, before you ask, I know this proof is silly.
                        $endgroup$
                        – Trevor Wilson
                        Mar 8 '13 at 2:21




                        $begingroup$
                        And yes, before you ask, I know this proof is silly.
                        $endgroup$
                        – Trevor Wilson
                        Mar 8 '13 at 2:21




                        2




                        2




                        $begingroup$
                        I think this proof conceptually connects many things, including well-ordering, order theory and so on. So, I really benefited.
                        $endgroup$
                        – user64066
                        Oct 22 '13 at 20:19




                        $begingroup$
                        I think this proof conceptually connects many things, including well-ordering, order theory and so on. So, I really benefited.
                        $endgroup$
                        – user64066
                        Oct 22 '13 at 20:19












                        $begingroup$
                        @user64066 Thanks, I'm glad to hear it.
                        $endgroup$
                        – Trevor Wilson
                        Oct 22 '13 at 20:30




                        $begingroup$
                        @user64066 Thanks, I'm glad to hear it.
                        $endgroup$
                        – Trevor Wilson
                        Oct 22 '13 at 20:30











                        4












                        $begingroup$

                        Let $G$ be a nonempty open set in $mathbb{R}$. Write $asim b$ if the closed interval $[a, b]$ or $[b, a]$ if $b<a$, lies in $G$.This is an equivalence relation, in particular $asim a$ since ${a}$ is itself a closed interval. $G$ is therefore the union of disjoint equivalence classes.



                        Let $C(a)$ be the equivalence class containing $a$. Then $C(a)$ is clearly an interval. Also $C(a)$ is open, for if $kin C(a)$, then $(k-epsilon, k+epsilon)subseteq G$ for sfficiently small $epsilon$.



                        But then $(k-epsilon, k+epsilon)subseteq C(a)$; so $G$ is the union of disjoint intervals. These are at most countable in number by Lindel$ddot{rm o}$f's theorem. This completes the proof.



                        Reference:G. De Barra, Measure theory and Integration, Horwood Publishing.



                        (http://infoedu.ir/wp-content/uploads/2014/03/MeasureTheoryBook.pdf)






                        share|cite|improve this answer











                        $endgroup$


















                          4












                          $begingroup$

                          Let $G$ be a nonempty open set in $mathbb{R}$. Write $asim b$ if the closed interval $[a, b]$ or $[b, a]$ if $b<a$, lies in $G$.This is an equivalence relation, in particular $asim a$ since ${a}$ is itself a closed interval. $G$ is therefore the union of disjoint equivalence classes.



                          Let $C(a)$ be the equivalence class containing $a$. Then $C(a)$ is clearly an interval. Also $C(a)$ is open, for if $kin C(a)$, then $(k-epsilon, k+epsilon)subseteq G$ for sfficiently small $epsilon$.



                          But then $(k-epsilon, k+epsilon)subseteq C(a)$; so $G$ is the union of disjoint intervals. These are at most countable in number by Lindel$ddot{rm o}$f's theorem. This completes the proof.



                          Reference:G. De Barra, Measure theory and Integration, Horwood Publishing.



                          (http://infoedu.ir/wp-content/uploads/2014/03/MeasureTheoryBook.pdf)






                          share|cite|improve this answer











                          $endgroup$
















                            4












                            4








                            4





                            $begingroup$

                            Let $G$ be a nonempty open set in $mathbb{R}$. Write $asim b$ if the closed interval $[a, b]$ or $[b, a]$ if $b<a$, lies in $G$.This is an equivalence relation, in particular $asim a$ since ${a}$ is itself a closed interval. $G$ is therefore the union of disjoint equivalence classes.



                            Let $C(a)$ be the equivalence class containing $a$. Then $C(a)$ is clearly an interval. Also $C(a)$ is open, for if $kin C(a)$, then $(k-epsilon, k+epsilon)subseteq G$ for sfficiently small $epsilon$.



                            But then $(k-epsilon, k+epsilon)subseteq C(a)$; so $G$ is the union of disjoint intervals. These are at most countable in number by Lindel$ddot{rm o}$f's theorem. This completes the proof.



                            Reference:G. De Barra, Measure theory and Integration, Horwood Publishing.



                            (http://infoedu.ir/wp-content/uploads/2014/03/MeasureTheoryBook.pdf)






                            share|cite|improve this answer











                            $endgroup$



                            Let $G$ be a nonempty open set in $mathbb{R}$. Write $asim b$ if the closed interval $[a, b]$ or $[b, a]$ if $b<a$, lies in $G$.This is an equivalence relation, in particular $asim a$ since ${a}$ is itself a closed interval. $G$ is therefore the union of disjoint equivalence classes.



                            Let $C(a)$ be the equivalence class containing $a$. Then $C(a)$ is clearly an interval. Also $C(a)$ is open, for if $kin C(a)$, then $(k-epsilon, k+epsilon)subseteq G$ for sfficiently small $epsilon$.



                            But then $(k-epsilon, k+epsilon)subseteq C(a)$; so $G$ is the union of disjoint intervals. These are at most countable in number by Lindel$ddot{rm o}$f's theorem. This completes the proof.



                            Reference:G. De Barra, Measure theory and Integration, Horwood Publishing.



                            (http://infoedu.ir/wp-content/uploads/2014/03/MeasureTheoryBook.pdf)







                            share|cite|improve this answer














                            share|cite|improve this answer



                            share|cite|improve this answer








                            edited Feb 16 '15 at 12:59


























                            community wiki





                            2 revs
                            sachin srivastava
























                                4












                                $begingroup$

                                $mathbb{R}$ with standard topology is second-countable space.



                                For a second-countable space with a (not necessarily countable) base, any open set can be written as a countable union of basic open set.



                                Given any base for a second countable space, is every open set the countable union of basic open sets?



                                Clearly, collection of open intervals is a base for the standard topology.
                                Hence any open set in $mathbb{R}$ can be written as countable union of open intervals.



                                If any two of exploited open intervals overlap, merge them. Then we have disjoint union of open intervals, which is still countable.






                                share|cite|improve this answer











                                $endgroup$


















                                  4












                                  $begingroup$

                                  $mathbb{R}$ with standard topology is second-countable space.



                                  For a second-countable space with a (not necessarily countable) base, any open set can be written as a countable union of basic open set.



                                  Given any base for a second countable space, is every open set the countable union of basic open sets?



                                  Clearly, collection of open intervals is a base for the standard topology.
                                  Hence any open set in $mathbb{R}$ can be written as countable union of open intervals.



                                  If any two of exploited open intervals overlap, merge them. Then we have disjoint union of open intervals, which is still countable.






                                  share|cite|improve this answer











                                  $endgroup$
















                                    4












                                    4








                                    4





                                    $begingroup$

                                    $mathbb{R}$ with standard topology is second-countable space.



                                    For a second-countable space with a (not necessarily countable) base, any open set can be written as a countable union of basic open set.



                                    Given any base for a second countable space, is every open set the countable union of basic open sets?



                                    Clearly, collection of open intervals is a base for the standard topology.
                                    Hence any open set in $mathbb{R}$ can be written as countable union of open intervals.



                                    If any two of exploited open intervals overlap, merge them. Then we have disjoint union of open intervals, which is still countable.






                                    share|cite|improve this answer











                                    $endgroup$



                                    $mathbb{R}$ with standard topology is second-countable space.



                                    For a second-countable space with a (not necessarily countable) base, any open set can be written as a countable union of basic open set.



                                    Given any base for a second countable space, is every open set the countable union of basic open sets?



                                    Clearly, collection of open intervals is a base for the standard topology.
                                    Hence any open set in $mathbb{R}$ can be written as countable union of open intervals.



                                    If any two of exploited open intervals overlap, merge them. Then we have disjoint union of open intervals, which is still countable.







                                    share|cite|improve this answer














                                    share|cite|improve this answer



                                    share|cite|improve this answer








                                    edited Apr 13 '17 at 12:19


























                                    community wiki





                                    2 revs
                                    Guldam
























                                        4












                                        $begingroup$

                                        This proof is an extended version of the nice proof proposed by
                                        Stromael and it serves best for beginners who want to understand every detail(that one that for any established mathematician logically seems trivial) of the proof.



                                        $ textbf{Proof:} $



                                        Let $U subseteq R$ be open and let $x in U$. Then Either $x$ is rational or $x$ is irrational.



                                        Suppose $x$ is rational, then define



                                        begin{align} I_x = bigcuplimits_{substack{Itext{ an open interval} \ x~in~I~subseteq~U}} I,end{align}



                                        Claim: $I_x$ is interval, $I_x$ is open and $ I_x subseteq U $



                                        Definition: An interval is a subset $ I subseteq R$ such that, for all $ a<c<b$ in $R$, if $ a,b in I $ then $ c in I$.



                                        Now, consider any $ a<c<b $ such that $ a,b in I_x$. We want to show that $ c in I_x $.



                                        Denote $I_a $ to be an interval such that $ x in I_a $ and $ a in I_a $. In other words $ I_a $ is one of the intervals from the union $ I_x $ that contains $a$. In the same way, let $ I_b $ be the interval such that $ x in I_b $ and $ b in I_b $.




                                        1. $ c=x $: If $c=x$ then by construction of $I_x$, $ x in I_x$


                                        2. $ c<x $: If $c<x$ then we have that either $ a<c<x<b $ or $ a<c<b<x $. Since $ x in I $ for every open interval $I$ of the union $I_x$ (by construction of $I_x$ ), we have that $x in I_a $ and $ x in I_b$. Since $ x in I_a $ then because $ I_a $ is an interval $ c in I_a$ and hence $ c in I_x $. And since $ x in I_b $ then because $ I_b $ is an interval $ c in I_b $ and hence $ c in I_x $. Thus, we concluded that $ c in I_x $.


                                        3. $ c > x $: If $ c>x $ then we have that either $ a<x<c<b $ or $ x<a<c<b $. Since $ x in I $ for every open interval $I$ of the union $I_x$ (by construction of $I_x$ ), we have that $x in I_a $ and $ x in I_b$. Since $ x in I_b $ then because $ I_b $ is an interval $ c in I_b $ and hence $ c in I_x $. As for the second case, note that since $ x in I_b$ we have that $ a in I_b $. But then, because $ I_b $ is an interval we have that $ c in I_b $ and hence $ c in I_x$. Hence we concluded that $ c in I_x $.



                                        This Proves that $ I_x $ is an interval.



                                        $ I_x $ is open because it is union of open sets.



                                        $ I_x subseteq U $ by construction.



                                        Suppose $x$ is irrational, then by openness of $ U $ there is $varepsilon > 0$ such that $(x - varepsilon, x + varepsilon) subseteq U$, and by the property of real numbers that for any irrational number there exists a sequence of rational unmbers that converges to that irrational number, there exists rational $y in (x - varepsilon, x + varepsilon) $. Then by construction $ (x - varepsilon, x + varepsilon) subseteq I_y $. Hence $x in I_y$. So any $x in U$ is in $I_q$ for some $q in U cap mathbb{Q}$, and so



                                        begin{align}U subseteq bigcuplimits_{q~in~U cap~mathbb{Q}} I_q.end{align}



                                        But $I_q subseteq U$ for each $q in U cap mathbb{Q}$; thus



                                        begin{align}U = bigcuplimits_{q~in~U cap~mathbb{Q}} I_q, end{align}



                                        which is a countable union of open intervals.



                                        Now let's show that intervals $ {I_q } ~ q in U cap mathbb{Q} $ are disjoint. Suppose there is $ i, j, in U cap mathbb{Q} $ such that $ I_i cap I_j neq emptyset $ then $ I_i subseteq I_q $ and $ I_j subseteq I_q $ for some $ q in U cap mathbb{Q} $



                                        Hence we constructed disjoint intervals $ {I_q } ~ q in U cap mathbb{Q} $ that are enumerated by rational numbers in $U$ and whose union is $U$. Since any subset of rational numbers is countable, $ {I_q } ~ q in U cap mathbb{Q} $ is countable as well. This finishes the proof.






                                        share|cite|improve this answer











                                        $endgroup$









                                        • 1




                                          $begingroup$
                                          What is the guarantee of the statement 'Denote $I_a $ to be an interval such that $ x in I_a $ '?
                                          $endgroup$
                                          – Unknown x
                                          Dec 30 '18 at 14:47










                                        • $begingroup$
                                          Also to me the first part of this answer seemed obscure, I tried to clarify it here math.stackexchange.com/questions/3040319/…
                                          $endgroup$
                                          – Jack J.
                                          Jan 17 at 9:28
















                                        4












                                        $begingroup$

                                        This proof is an extended version of the nice proof proposed by
                                        Stromael and it serves best for beginners who want to understand every detail(that one that for any established mathematician logically seems trivial) of the proof.



                                        $ textbf{Proof:} $



                                        Let $U subseteq R$ be open and let $x in U$. Then Either $x$ is rational or $x$ is irrational.



                                        Suppose $x$ is rational, then define



                                        begin{align} I_x = bigcuplimits_{substack{Itext{ an open interval} \ x~in~I~subseteq~U}} I,end{align}



                                        Claim: $I_x$ is interval, $I_x$ is open and $ I_x subseteq U $



                                        Definition: An interval is a subset $ I subseteq R$ such that, for all $ a<c<b$ in $R$, if $ a,b in I $ then $ c in I$.



                                        Now, consider any $ a<c<b $ such that $ a,b in I_x$. We want to show that $ c in I_x $.



                                        Denote $I_a $ to be an interval such that $ x in I_a $ and $ a in I_a $. In other words $ I_a $ is one of the intervals from the union $ I_x $ that contains $a$. In the same way, let $ I_b $ be the interval such that $ x in I_b $ and $ b in I_b $.




                                        1. $ c=x $: If $c=x$ then by construction of $I_x$, $ x in I_x$


                                        2. $ c<x $: If $c<x$ then we have that either $ a<c<x<b $ or $ a<c<b<x $. Since $ x in I $ for every open interval $I$ of the union $I_x$ (by construction of $I_x$ ), we have that $x in I_a $ and $ x in I_b$. Since $ x in I_a $ then because $ I_a $ is an interval $ c in I_a$ and hence $ c in I_x $. And since $ x in I_b $ then because $ I_b $ is an interval $ c in I_b $ and hence $ c in I_x $. Thus, we concluded that $ c in I_x $.


                                        3. $ c > x $: If $ c>x $ then we have that either $ a<x<c<b $ or $ x<a<c<b $. Since $ x in I $ for every open interval $I$ of the union $I_x$ (by construction of $I_x$ ), we have that $x in I_a $ and $ x in I_b$. Since $ x in I_b $ then because $ I_b $ is an interval $ c in I_b $ and hence $ c in I_x $. As for the second case, note that since $ x in I_b$ we have that $ a in I_b $. But then, because $ I_b $ is an interval we have that $ c in I_b $ and hence $ c in I_x$. Hence we concluded that $ c in I_x $.



                                        This Proves that $ I_x $ is an interval.



                                        $ I_x $ is open because it is union of open sets.



                                        $ I_x subseteq U $ by construction.



                                        Suppose $x$ is irrational, then by openness of $ U $ there is $varepsilon > 0$ such that $(x - varepsilon, x + varepsilon) subseteq U$, and by the property of real numbers that for any irrational number there exists a sequence of rational unmbers that converges to that irrational number, there exists rational $y in (x - varepsilon, x + varepsilon) $. Then by construction $ (x - varepsilon, x + varepsilon) subseteq I_y $. Hence $x in I_y$. So any $x in U$ is in $I_q$ for some $q in U cap mathbb{Q}$, and so



                                        begin{align}U subseteq bigcuplimits_{q~in~U cap~mathbb{Q}} I_q.end{align}



                                        But $I_q subseteq U$ for each $q in U cap mathbb{Q}$; thus



                                        begin{align}U = bigcuplimits_{q~in~U cap~mathbb{Q}} I_q, end{align}



                                        which is a countable union of open intervals.



                                        Now let's show that intervals $ {I_q } ~ q in U cap mathbb{Q} $ are disjoint. Suppose there is $ i, j, in U cap mathbb{Q} $ such that $ I_i cap I_j neq emptyset $ then $ I_i subseteq I_q $ and $ I_j subseteq I_q $ for some $ q in U cap mathbb{Q} $



                                        Hence we constructed disjoint intervals $ {I_q } ~ q in U cap mathbb{Q} $ that are enumerated by rational numbers in $U$ and whose union is $U$. Since any subset of rational numbers is countable, $ {I_q } ~ q in U cap mathbb{Q} $ is countable as well. This finishes the proof.






                                        share|cite|improve this answer











                                        $endgroup$









                                        • 1




                                          $begingroup$
                                          What is the guarantee of the statement 'Denote $I_a $ to be an interval such that $ x in I_a $ '?
                                          $endgroup$
                                          – Unknown x
                                          Dec 30 '18 at 14:47










                                        • $begingroup$
                                          Also to me the first part of this answer seemed obscure, I tried to clarify it here math.stackexchange.com/questions/3040319/…
                                          $endgroup$
                                          – Jack J.
                                          Jan 17 at 9:28














                                        4












                                        4








                                        4





                                        $begingroup$

                                        This proof is an extended version of the nice proof proposed by
                                        Stromael and it serves best for beginners who want to understand every detail(that one that for any established mathematician logically seems trivial) of the proof.



                                        $ textbf{Proof:} $



                                        Let $U subseteq R$ be open and let $x in U$. Then Either $x$ is rational or $x$ is irrational.



                                        Suppose $x$ is rational, then define



                                        begin{align} I_x = bigcuplimits_{substack{Itext{ an open interval} \ x~in~I~subseteq~U}} I,end{align}



                                        Claim: $I_x$ is interval, $I_x$ is open and $ I_x subseteq U $



                                        Definition: An interval is a subset $ I subseteq R$ such that, for all $ a<c<b$ in $R$, if $ a,b in I $ then $ c in I$.



                                        Now, consider any $ a<c<b $ such that $ a,b in I_x$. We want to show that $ c in I_x $.



                                        Denote $I_a $ to be an interval such that $ x in I_a $ and $ a in I_a $. In other words $ I_a $ is one of the intervals from the union $ I_x $ that contains $a$. In the same way, let $ I_b $ be the interval such that $ x in I_b $ and $ b in I_b $.




                                        1. $ c=x $: If $c=x$ then by construction of $I_x$, $ x in I_x$


                                        2. $ c<x $: If $c<x$ then we have that either $ a<c<x<b $ or $ a<c<b<x $. Since $ x in I $ for every open interval $I$ of the union $I_x$ (by construction of $I_x$ ), we have that $x in I_a $ and $ x in I_b$. Since $ x in I_a $ then because $ I_a $ is an interval $ c in I_a$ and hence $ c in I_x $. And since $ x in I_b $ then because $ I_b $ is an interval $ c in I_b $ and hence $ c in I_x $. Thus, we concluded that $ c in I_x $.


                                        3. $ c > x $: If $ c>x $ then we have that either $ a<x<c<b $ or $ x<a<c<b $. Since $ x in I $ for every open interval $I$ of the union $I_x$ (by construction of $I_x$ ), we have that $x in I_a $ and $ x in I_b$. Since $ x in I_b $ then because $ I_b $ is an interval $ c in I_b $ and hence $ c in I_x $. As for the second case, note that since $ x in I_b$ we have that $ a in I_b $. But then, because $ I_b $ is an interval we have that $ c in I_b $ and hence $ c in I_x$. Hence we concluded that $ c in I_x $.



                                        This Proves that $ I_x $ is an interval.



                                        $ I_x $ is open because it is union of open sets.



                                        $ I_x subseteq U $ by construction.



                                        Suppose $x$ is irrational, then by openness of $ U $ there is $varepsilon > 0$ such that $(x - varepsilon, x + varepsilon) subseteq U$, and by the property of real numbers that for any irrational number there exists a sequence of rational unmbers that converges to that irrational number, there exists rational $y in (x - varepsilon, x + varepsilon) $. Then by construction $ (x - varepsilon, x + varepsilon) subseteq I_y $. Hence $x in I_y$. So any $x in U$ is in $I_q$ for some $q in U cap mathbb{Q}$, and so



                                        begin{align}U subseteq bigcuplimits_{q~in~U cap~mathbb{Q}} I_q.end{align}



                                        But $I_q subseteq U$ for each $q in U cap mathbb{Q}$; thus



                                        begin{align}U = bigcuplimits_{q~in~U cap~mathbb{Q}} I_q, end{align}



                                        which is a countable union of open intervals.



                                        Now let's show that intervals $ {I_q } ~ q in U cap mathbb{Q} $ are disjoint. Suppose there is $ i, j, in U cap mathbb{Q} $ such that $ I_i cap I_j neq emptyset $ then $ I_i subseteq I_q $ and $ I_j subseteq I_q $ for some $ q in U cap mathbb{Q} $



                                        Hence we constructed disjoint intervals $ {I_q } ~ q in U cap mathbb{Q} $ that are enumerated by rational numbers in $U$ and whose union is $U$. Since any subset of rational numbers is countable, $ {I_q } ~ q in U cap mathbb{Q} $ is countable as well. This finishes the proof.






                                        share|cite|improve this answer











                                        $endgroup$



                                        This proof is an extended version of the nice proof proposed by
                                        Stromael and it serves best for beginners who want to understand every detail(that one that for any established mathematician logically seems trivial) of the proof.



                                        $ textbf{Proof:} $



                                        Let $U subseteq R$ be open and let $x in U$. Then Either $x$ is rational or $x$ is irrational.



                                        Suppose $x$ is rational, then define



                                        begin{align} I_x = bigcuplimits_{substack{Itext{ an open interval} \ x~in~I~subseteq~U}} I,end{align}



                                        Claim: $I_x$ is interval, $I_x$ is open and $ I_x subseteq U $



                                        Definition: An interval is a subset $ I subseteq R$ such that, for all $ a<c<b$ in $R$, if $ a,b in I $ then $ c in I$.



                                        Now, consider any $ a<c<b $ such that $ a,b in I_x$. We want to show that $ c in I_x $.



                                        Denote $I_a $ to be an interval such that $ x in I_a $ and $ a in I_a $. In other words $ I_a $ is one of the intervals from the union $ I_x $ that contains $a$. In the same way, let $ I_b $ be the interval such that $ x in I_b $ and $ b in I_b $.




                                        1. $ c=x $: If $c=x$ then by construction of $I_x$, $ x in I_x$


                                        2. $ c<x $: If $c<x$ then we have that either $ a<c<x<b $ or $ a<c<b<x $. Since $ x in I $ for every open interval $I$ of the union $I_x$ (by construction of $I_x$ ), we have that $x in I_a $ and $ x in I_b$. Since $ x in I_a $ then because $ I_a $ is an interval $ c in I_a$ and hence $ c in I_x $. And since $ x in I_b $ then because $ I_b $ is an interval $ c in I_b $ and hence $ c in I_x $. Thus, we concluded that $ c in I_x $.


                                        3. $ c > x $: If $ c>x $ then we have that either $ a<x<c<b $ or $ x<a<c<b $. Since $ x in I $ for every open interval $I$ of the union $I_x$ (by construction of $I_x$ ), we have that $x in I_a $ and $ x in I_b$. Since $ x in I_b $ then because $ I_b $ is an interval $ c in I_b $ and hence $ c in I_x $. As for the second case, note that since $ x in I_b$ we have that $ a in I_b $. But then, because $ I_b $ is an interval we have that $ c in I_b $ and hence $ c in I_x$. Hence we concluded that $ c in I_x $.



                                        This Proves that $ I_x $ is an interval.



                                        $ I_x $ is open because it is union of open sets.



                                        $ I_x subseteq U $ by construction.



                                        Suppose $x$ is irrational, then by openness of $ U $ there is $varepsilon > 0$ such that $(x - varepsilon, x + varepsilon) subseteq U$, and by the property of real numbers that for any irrational number there exists a sequence of rational unmbers that converges to that irrational number, there exists rational $y in (x - varepsilon, x + varepsilon) $. Then by construction $ (x - varepsilon, x + varepsilon) subseteq I_y $. Hence $x in I_y$. So any $x in U$ is in $I_q$ for some $q in U cap mathbb{Q}$, and so



                                        begin{align}U subseteq bigcuplimits_{q~in~U cap~mathbb{Q}} I_q.end{align}



                                        But $I_q subseteq U$ for each $q in U cap mathbb{Q}$; thus



                                        begin{align}U = bigcuplimits_{q~in~U cap~mathbb{Q}} I_q, end{align}



                                        which is a countable union of open intervals.



                                        Now let's show that intervals $ {I_q } ~ q in U cap mathbb{Q} $ are disjoint. Suppose there is $ i, j, in U cap mathbb{Q} $ such that $ I_i cap I_j neq emptyset $ then $ I_i subseteq I_q $ and $ I_j subseteq I_q $ for some $ q in U cap mathbb{Q} $



                                        Hence we constructed disjoint intervals $ {I_q } ~ q in U cap mathbb{Q} $ that are enumerated by rational numbers in $U$ and whose union is $U$. Since any subset of rational numbers is countable, $ {I_q } ~ q in U cap mathbb{Q} $ is countable as well. This finishes the proof.







                                        share|cite|improve this answer














                                        share|cite|improve this answer



                                        share|cite|improve this answer








                                        edited Dec 30 '18 at 14:23


























                                        community wiki





                                        2 revs, 2 users 99%
                                        G.T.









                                        • 1




                                          $begingroup$
                                          What is the guarantee of the statement 'Denote $I_a $ to be an interval such that $ x in I_a $ '?
                                          $endgroup$
                                          – Unknown x
                                          Dec 30 '18 at 14:47










                                        • $begingroup$
                                          Also to me the first part of this answer seemed obscure, I tried to clarify it here math.stackexchange.com/questions/3040319/…
                                          $endgroup$
                                          – Jack J.
                                          Jan 17 at 9:28














                                        • 1




                                          $begingroup$
                                          What is the guarantee of the statement 'Denote $I_a $ to be an interval such that $ x in I_a $ '?
                                          $endgroup$
                                          – Unknown x
                                          Dec 30 '18 at 14:47










                                        • $begingroup$
                                          Also to me the first part of this answer seemed obscure, I tried to clarify it here math.stackexchange.com/questions/3040319/…
                                          $endgroup$
                                          – Jack J.
                                          Jan 17 at 9:28








                                        1




                                        1




                                        $begingroup$
                                        What is the guarantee of the statement 'Denote $I_a $ to be an interval such that $ x in I_a $ '?
                                        $endgroup$
                                        – Unknown x
                                        Dec 30 '18 at 14:47




                                        $begingroup$
                                        What is the guarantee of the statement 'Denote $I_a $ to be an interval such that $ x in I_a $ '?
                                        $endgroup$
                                        – Unknown x
                                        Dec 30 '18 at 14:47












                                        $begingroup$
                                        Also to me the first part of this answer seemed obscure, I tried to clarify it here math.stackexchange.com/questions/3040319/…
                                        $endgroup$
                                        – Jack J.
                                        Jan 17 at 9:28




                                        $begingroup$
                                        Also to me the first part of this answer seemed obscure, I tried to clarify it here math.stackexchange.com/questions/3040319/…
                                        $endgroup$
                                        – Jack J.
                                        Jan 17 at 9:28











                                        3












                                        $begingroup$

                                        I hope that this is right as this is a lemma i've thought of and i plan to use in a project due in several days and it somewhat generalizes the question asked:



                                        Suppose that $U$ is a set of intervals in $mathbb{R}$ (closed, open, semi-closed, etc.). Then there exists a set of disjoint intervals $V$ in $mathbb{R}$ s.t. $bigcup_{Iin U}I=biguplus_{Iin V}I$. If non of the intervals are degenerate or $U$ is countable, then this set can be taken to be countable. And if they were all open, then we could take the segments of $V$ to be open (And also, if $U$ is countable then we won't be needing the Axiom of Choice).



                                        proof:
                                        Let us order the elements of $U$: $U=langle I_beta,|,betaleqalpharangle$ where $alpha$ is the first ordinal of cardinality $|U|$. (If $U$ is countable then this doesn't require AC and from here on will be standard induction with a simple construction in the end for $omega$).



                                        I'll build $V_beta=langle J^gamma_beta,|,gammaleqbetarangle$ - a sequence of segments for all $betaleqalpha$ such that every two sets in $V_beta$ are either disjoint or equal and such that $displaystyle{bigcup_{gammaleqbeta}I_gamma=biguplus_{gammaleqbeta}J^gamma_beta}$ and $forallbeta$, $langle J^beta_gammarangle_{gammageqbeta}$ is a non-descending sequence of sets ,by means of transfinite induction.



                                        For $V_0$ take, $V_0=langle I_0rangle$. Suppose that we have built the required $V_gamma$, $gamma<beta$ for some $betaleqalpha$, then we will build $V_beta$ in the following way: $forallgamma<beta$, denote $widetilde{J}_gamma$=$bigcup_{gammaleqdelta<beta}J_delta^gamma$-still segments (non-decreasing sequence). If $I_beta$ is disjoint of all $widetilde{J}_gamma$, taking $V_beta!=!langle widetilde{J}_gamma,|,gamma<betaranglecup{(beta,I_beta)}$ would give us a sequence $langle V_gamma,|,gammaleqbetarangle$ satisfying the required conditions of it (the only non trivial thing is that pairs of $widetilde{J}_gamma$ are either disjoint of each other or are equal, but that is also quite trivial since if the contrary would have occurred, then $existsgamma_1<gamma_2<beta$ s.t. $widetilde{J}_{gamma_1}neqwidetilde{J}_{gamma_2}$ and $widetilde{J}_{gamma_1}capwidetilde{J}_{gamma_2}neqemptyset$, but then, $exists beta>delta_1geqgamma_1, beta>delta_2geqgamma_2$ s.t. $J^{gamma_1}_{delta_1}cap J^{gamma_2}_{delta_2}neqemptyset$, meaning that either $J^{gamma_1}_{delta_2}= J^{gamma_2}_{delta_2}$ or $J^{gamma_1}_{delta_1}= J^{gamma_2}_{delta_1}$ thus, $forallbeta>epsilongeqdelta_1,delta_2$, $J^{gamma_1}_{epsilon}= J^{gamma_2}_{epsilon}$ and since we are talking here about non-decreasing sequences, this will contradict $widetilde{J}_{gamma_1}neqwidetilde{J}_{gamma_2}$). And if $I_beta$ isn't disjoint of all $widetilde{J}_gamma$, Then we can take $J_beta^gamma=widetilde{J}_gamma$ for all $gamma<beta$ that don't intersect with $I_beta$ and $J_beta^gamma=bigcup_{delta<betatext{ s.t. }widetilde{J}_deltacap I_betaneqemptyset}{widetilde{J}_delta}cup I_beta$ - segment for all of the other $gammaleqbeta$. Then again from the same arguments, $langle V_gamma,|,gammaleqbetarangle$ would satisfy the required conditions.



                                        Finally, we can take $V={J_alpha^beta,|,betaleqalpha}$ to get what we wanted in the first place. And obviously, if our segments were all non-degenerate to begin with, from the way we constructed our set, all of the segments in $V$ will be non-degenerate (and thus of positive measure), but they are disjoint and so there is only a countable number of them. And if the segments in $U$ were all open, then obviously, so will the segments in $V$.$square$






                                        share|cite|improve this answer











                                        $endgroup$


















                                          3












                                          $begingroup$

                                          I hope that this is right as this is a lemma i've thought of and i plan to use in a project due in several days and it somewhat generalizes the question asked:



                                          Suppose that $U$ is a set of intervals in $mathbb{R}$ (closed, open, semi-closed, etc.). Then there exists a set of disjoint intervals $V$ in $mathbb{R}$ s.t. $bigcup_{Iin U}I=biguplus_{Iin V}I$. If non of the intervals are degenerate or $U$ is countable, then this set can be taken to be countable. And if they were all open, then we could take the segments of $V$ to be open (And also, if $U$ is countable then we won't be needing the Axiom of Choice).



                                          proof:
                                          Let us order the elements of $U$: $U=langle I_beta,|,betaleqalpharangle$ where $alpha$ is the first ordinal of cardinality $|U|$. (If $U$ is countable then this doesn't require AC and from here on will be standard induction with a simple construction in the end for $omega$).



                                          I'll build $V_beta=langle J^gamma_beta,|,gammaleqbetarangle$ - a sequence of segments for all $betaleqalpha$ such that every two sets in $V_beta$ are either disjoint or equal and such that $displaystyle{bigcup_{gammaleqbeta}I_gamma=biguplus_{gammaleqbeta}J^gamma_beta}$ and $forallbeta$, $langle J^beta_gammarangle_{gammageqbeta}$ is a non-descending sequence of sets ,by means of transfinite induction.



                                          For $V_0$ take, $V_0=langle I_0rangle$. Suppose that we have built the required $V_gamma$, $gamma<beta$ for some $betaleqalpha$, then we will build $V_beta$ in the following way: $forallgamma<beta$, denote $widetilde{J}_gamma$=$bigcup_{gammaleqdelta<beta}J_delta^gamma$-still segments (non-decreasing sequence). If $I_beta$ is disjoint of all $widetilde{J}_gamma$, taking $V_beta!=!langle widetilde{J}_gamma,|,gamma<betaranglecup{(beta,I_beta)}$ would give us a sequence $langle V_gamma,|,gammaleqbetarangle$ satisfying the required conditions of it (the only non trivial thing is that pairs of $widetilde{J}_gamma$ are either disjoint of each other or are equal, but that is also quite trivial since if the contrary would have occurred, then $existsgamma_1<gamma_2<beta$ s.t. $widetilde{J}_{gamma_1}neqwidetilde{J}_{gamma_2}$ and $widetilde{J}_{gamma_1}capwidetilde{J}_{gamma_2}neqemptyset$, but then, $exists beta>delta_1geqgamma_1, beta>delta_2geqgamma_2$ s.t. $J^{gamma_1}_{delta_1}cap J^{gamma_2}_{delta_2}neqemptyset$, meaning that either $J^{gamma_1}_{delta_2}= J^{gamma_2}_{delta_2}$ or $J^{gamma_1}_{delta_1}= J^{gamma_2}_{delta_1}$ thus, $forallbeta>epsilongeqdelta_1,delta_2$, $J^{gamma_1}_{epsilon}= J^{gamma_2}_{epsilon}$ and since we are talking here about non-decreasing sequences, this will contradict $widetilde{J}_{gamma_1}neqwidetilde{J}_{gamma_2}$). And if $I_beta$ isn't disjoint of all $widetilde{J}_gamma$, Then we can take $J_beta^gamma=widetilde{J}_gamma$ for all $gamma<beta$ that don't intersect with $I_beta$ and $J_beta^gamma=bigcup_{delta<betatext{ s.t. }widetilde{J}_deltacap I_betaneqemptyset}{widetilde{J}_delta}cup I_beta$ - segment for all of the other $gammaleqbeta$. Then again from the same arguments, $langle V_gamma,|,gammaleqbetarangle$ would satisfy the required conditions.



                                          Finally, we can take $V={J_alpha^beta,|,betaleqalpha}$ to get what we wanted in the first place. And obviously, if our segments were all non-degenerate to begin with, from the way we constructed our set, all of the segments in $V$ will be non-degenerate (and thus of positive measure), but they are disjoint and so there is only a countable number of them. And if the segments in $U$ were all open, then obviously, so will the segments in $V$.$square$






                                          share|cite|improve this answer











                                          $endgroup$
















                                            3












                                            3








                                            3





                                            $begingroup$

                                            I hope that this is right as this is a lemma i've thought of and i plan to use in a project due in several days and it somewhat generalizes the question asked:



                                            Suppose that $U$ is a set of intervals in $mathbb{R}$ (closed, open, semi-closed, etc.). Then there exists a set of disjoint intervals $V$ in $mathbb{R}$ s.t. $bigcup_{Iin U}I=biguplus_{Iin V}I$. If non of the intervals are degenerate or $U$ is countable, then this set can be taken to be countable. And if they were all open, then we could take the segments of $V$ to be open (And also, if $U$ is countable then we won't be needing the Axiom of Choice).



                                            proof:
                                            Let us order the elements of $U$: $U=langle I_beta,|,betaleqalpharangle$ where $alpha$ is the first ordinal of cardinality $|U|$. (If $U$ is countable then this doesn't require AC and from here on will be standard induction with a simple construction in the end for $omega$).



                                            I'll build $V_beta=langle J^gamma_beta,|,gammaleqbetarangle$ - a sequence of segments for all $betaleqalpha$ such that every two sets in $V_beta$ are either disjoint or equal and such that $displaystyle{bigcup_{gammaleqbeta}I_gamma=biguplus_{gammaleqbeta}J^gamma_beta}$ and $forallbeta$, $langle J^beta_gammarangle_{gammageqbeta}$ is a non-descending sequence of sets ,by means of transfinite induction.



                                            For $V_0$ take, $V_0=langle I_0rangle$. Suppose that we have built the required $V_gamma$, $gamma<beta$ for some $betaleqalpha$, then we will build $V_beta$ in the following way: $forallgamma<beta$, denote $widetilde{J}_gamma$=$bigcup_{gammaleqdelta<beta}J_delta^gamma$-still segments (non-decreasing sequence). If $I_beta$ is disjoint of all $widetilde{J}_gamma$, taking $V_beta!=!langle widetilde{J}_gamma,|,gamma<betaranglecup{(beta,I_beta)}$ would give us a sequence $langle V_gamma,|,gammaleqbetarangle$ satisfying the required conditions of it (the only non trivial thing is that pairs of $widetilde{J}_gamma$ are either disjoint of each other or are equal, but that is also quite trivial since if the contrary would have occurred, then $existsgamma_1<gamma_2<beta$ s.t. $widetilde{J}_{gamma_1}neqwidetilde{J}_{gamma_2}$ and $widetilde{J}_{gamma_1}capwidetilde{J}_{gamma_2}neqemptyset$, but then, $exists beta>delta_1geqgamma_1, beta>delta_2geqgamma_2$ s.t. $J^{gamma_1}_{delta_1}cap J^{gamma_2}_{delta_2}neqemptyset$, meaning that either $J^{gamma_1}_{delta_2}= J^{gamma_2}_{delta_2}$ or $J^{gamma_1}_{delta_1}= J^{gamma_2}_{delta_1}$ thus, $forallbeta>epsilongeqdelta_1,delta_2$, $J^{gamma_1}_{epsilon}= J^{gamma_2}_{epsilon}$ and since we are talking here about non-decreasing sequences, this will contradict $widetilde{J}_{gamma_1}neqwidetilde{J}_{gamma_2}$). And if $I_beta$ isn't disjoint of all $widetilde{J}_gamma$, Then we can take $J_beta^gamma=widetilde{J}_gamma$ for all $gamma<beta$ that don't intersect with $I_beta$ and $J_beta^gamma=bigcup_{delta<betatext{ s.t. }widetilde{J}_deltacap I_betaneqemptyset}{widetilde{J}_delta}cup I_beta$ - segment for all of the other $gammaleqbeta$. Then again from the same arguments, $langle V_gamma,|,gammaleqbetarangle$ would satisfy the required conditions.



                                            Finally, we can take $V={J_alpha^beta,|,betaleqalpha}$ to get what we wanted in the first place. And obviously, if our segments were all non-degenerate to begin with, from the way we constructed our set, all of the segments in $V$ will be non-degenerate (and thus of positive measure), but they are disjoint and so there is only a countable number of them. And if the segments in $U$ were all open, then obviously, so will the segments in $V$.$square$






                                            share|cite|improve this answer











                                            $endgroup$



                                            I hope that this is right as this is a lemma i've thought of and i plan to use in a project due in several days and it somewhat generalizes the question asked:



                                            Suppose that $U$ is a set of intervals in $mathbb{R}$ (closed, open, semi-closed, etc.). Then there exists a set of disjoint intervals $V$ in $mathbb{R}$ s.t. $bigcup_{Iin U}I=biguplus_{Iin V}I$. If non of the intervals are degenerate or $U$ is countable, then this set can be taken to be countable. And if they were all open, then we could take the segments of $V$ to be open (And also, if $U$ is countable then we won't be needing the Axiom of Choice).



                                            proof:
                                            Let us order the elements of $U$: $U=langle I_beta,|,betaleqalpharangle$ where $alpha$ is the first ordinal of cardinality $|U|$. (If $U$ is countable then this doesn't require AC and from here on will be standard induction with a simple construction in the end for $omega$).



                                            I'll build $V_beta=langle J^gamma_beta,|,gammaleqbetarangle$ - a sequence of segments for all $betaleqalpha$ such that every two sets in $V_beta$ are either disjoint or equal and such that $displaystyle{bigcup_{gammaleqbeta}I_gamma=biguplus_{gammaleqbeta}J^gamma_beta}$ and $forallbeta$, $langle J^beta_gammarangle_{gammageqbeta}$ is a non-descending sequence of sets ,by means of transfinite induction.



                                            For $V_0$ take, $V_0=langle I_0rangle$. Suppose that we have built the required $V_gamma$, $gamma<beta$ for some $betaleqalpha$, then we will build $V_beta$ in the following way: $forallgamma<beta$, denote $widetilde{J}_gamma$=$bigcup_{gammaleqdelta<beta}J_delta^gamma$-still segments (non-decreasing sequence). If $I_beta$ is disjoint of all $widetilde{J}_gamma$, taking $V_beta!=!langle widetilde{J}_gamma,|,gamma<betaranglecup{(beta,I_beta)}$ would give us a sequence $langle V_gamma,|,gammaleqbetarangle$ satisfying the required conditions of it (the only non trivial thing is that pairs of $widetilde{J}_gamma$ are either disjoint of each other or are equal, but that is also quite trivial since if the contrary would have occurred, then $existsgamma_1<gamma_2<beta$ s.t. $widetilde{J}_{gamma_1}neqwidetilde{J}_{gamma_2}$ and $widetilde{J}_{gamma_1}capwidetilde{J}_{gamma_2}neqemptyset$, but then, $exists beta>delta_1geqgamma_1, beta>delta_2geqgamma_2$ s.t. $J^{gamma_1}_{delta_1}cap J^{gamma_2}_{delta_2}neqemptyset$, meaning that either $J^{gamma_1}_{delta_2}= J^{gamma_2}_{delta_2}$ or $J^{gamma_1}_{delta_1}= J^{gamma_2}_{delta_1}$ thus, $forallbeta>epsilongeqdelta_1,delta_2$, $J^{gamma_1}_{epsilon}= J^{gamma_2}_{epsilon}$ and since we are talking here about non-decreasing sequences, this will contradict $widetilde{J}_{gamma_1}neqwidetilde{J}_{gamma_2}$). And if $I_beta$ isn't disjoint of all $widetilde{J}_gamma$, Then we can take $J_beta^gamma=widetilde{J}_gamma$ for all $gamma<beta$ that don't intersect with $I_beta$ and $J_beta^gamma=bigcup_{delta<betatext{ s.t. }widetilde{J}_deltacap I_betaneqemptyset}{widetilde{J}_delta}cup I_beta$ - segment for all of the other $gammaleqbeta$. Then again from the same arguments, $langle V_gamma,|,gammaleqbetarangle$ would satisfy the required conditions.



                                            Finally, we can take $V={J_alpha^beta,|,betaleqalpha}$ to get what we wanted in the first place. And obviously, if our segments were all non-degenerate to begin with, from the way we constructed our set, all of the segments in $V$ will be non-degenerate (and thus of positive measure), but they are disjoint and so there is only a countable number of them. And if the segments in $U$ were all open, then obviously, so will the segments in $V$.$square$







                                            share|cite|improve this answer














                                            share|cite|improve this answer



                                            share|cite|improve this answer








                                            answered Sep 5 '13 at 11:55


























                                            community wiki





                                            peter
























                                                3












                                                $begingroup$

                                                The balls with radii $frac{1}{n}$ and center at a rational number form a basis for the euclidean topology. This family is countable as it is a countable union of countable sets. We have found a countable basis, so we are done.






                                                share|cite|improve this answer











                                                $endgroup$









                                                • 3




                                                  $begingroup$
                                                  Ah, but the intervals are requested to be disjoint, and your intervals aren’t disjoint.
                                                  $endgroup$
                                                  – Lubin
                                                  Oct 8 '16 at 3:11
















                                                3












                                                $begingroup$

                                                The balls with radii $frac{1}{n}$ and center at a rational number form a basis for the euclidean topology. This family is countable as it is a countable union of countable sets. We have found a countable basis, so we are done.






                                                share|cite|improve this answer











                                                $endgroup$









                                                • 3




                                                  $begingroup$
                                                  Ah, but the intervals are requested to be disjoint, and your intervals aren’t disjoint.
                                                  $endgroup$
                                                  – Lubin
                                                  Oct 8 '16 at 3:11














                                                3












                                                3








                                                3





                                                $begingroup$

                                                The balls with radii $frac{1}{n}$ and center at a rational number form a basis for the euclidean topology. This family is countable as it is a countable union of countable sets. We have found a countable basis, so we are done.






                                                share|cite|improve this answer











                                                $endgroup$



                                                The balls with radii $frac{1}{n}$ and center at a rational number form a basis for the euclidean topology. This family is countable as it is a countable union of countable sets. We have found a countable basis, so we are done.







                                                share|cite|improve this answer














                                                share|cite|improve this answer



                                                share|cite|improve this answer








                                                answered Sep 20 '15 at 14:35


























                                                community wiki





                                                Jorge Fernández Hidalgo









                                                • 3




                                                  $begingroup$
                                                  Ah, but the intervals are requested to be disjoint, and your intervals aren’t disjoint.
                                                  $endgroup$
                                                  – Lubin
                                                  Oct 8 '16 at 3:11














                                                • 3




                                                  $begingroup$
                                                  Ah, but the intervals are requested to be disjoint, and your intervals aren’t disjoint.
                                                  $endgroup$
                                                  – Lubin
                                                  Oct 8 '16 at 3:11








                                                3




                                                3




                                                $begingroup$
                                                Ah, but the intervals are requested to be disjoint, and your intervals aren’t disjoint.
                                                $endgroup$
                                                – Lubin
                                                Oct 8 '16 at 3:11




                                                $begingroup$
                                                Ah, but the intervals are requested to be disjoint, and your intervals aren’t disjoint.
                                                $endgroup$
                                                – Lubin
                                                Oct 8 '16 at 3:11











                                                2












                                                $begingroup$

                                                Essentially nothing differs here from the two previous responses which rely principally on the fact that $mathbb{R} $ is locally connected. However I present here a proof that hopefully will feel accessible to readers with a slightly lower level of topological literary at the cost of appearing cumbersome to an expert.



                                                Consider the connected components of $U$.
                                                $U_x subseteq U$ is defined to be the connected component of U containing $x$ if $U_x$ is the largest connected subset of $U$ which contains $x$.
                                                Clearly by definition $U_x=U_v$ if $v in U_x$.
                                                Therefore if $U_a cap U_b neq varnothing$ then $U_a=U_b$.
                                                We see that ${U_x}_{xin U}$ is a disjoint collection.
                                                Also it should be clear that $bigcup limits_{xin U} U_x = U$.



                                                Now we show that $forall x$ $U_x$ is open.
                                                Let $yin U_x subseteq U$.
                                                Since $U$ is open there exists $epsilon>0$ such that $(y-epsilon, y+epsilon )subseteq U$.
                                                Sets of real numbers are connected iff they are intervals, singletons or empty.
                                                $(y-epsilon,y+epsilon)$ an interval hence it is connected.
                                                Therefore since $U_y$ is the largest connected subset of $U$ containing $y$ we must have $(y-epsilon,y+epsilon)subseteq U_y =U_x$.
                                                This shows that $U_x$ is open for all $x$.



                                                $U_x$ open and connected implies that $U_x$ must be an open interval.



                                                Also $mathbb{Q}$ dense in $mathbb{R}$,
                                                so $forall xin U$, $U_xcap mathbb{Q}neq varnothing$ and $U_x=U_q$ for some $qinmathbb{Q}$.
                                                So we can write ${U_x}_{xin U}={U_q}_{qin S}$ for some $Ssubseteq mathbb{Q}$.
                                                $mathbb{Q}$ is countable so $S$ is at most countable.



                                                In conclusion, we have just shown that the union of the connected components of $U$ is a disjoint union of open intervals that equals $U$ and is at most countable.






                                                share|cite|improve this answer











                                                $endgroup$


















                                                  2












                                                  $begingroup$

                                                  Essentially nothing differs here from the two previous responses which rely principally on the fact that $mathbb{R} $ is locally connected. However I present here a proof that hopefully will feel accessible to readers with a slightly lower level of topological literary at the cost of appearing cumbersome to an expert.



                                                  Consider the connected components of $U$.
                                                  $U_x subseteq U$ is defined to be the connected component of U containing $x$ if $U_x$ is the largest connected subset of $U$ which contains $x$.
                                                  Clearly by definition $U_x=U_v$ if $v in U_x$.
                                                  Therefore if $U_a cap U_b neq varnothing$ then $U_a=U_b$.
                                                  We see that ${U_x}_{xin U}$ is a disjoint collection.
                                                  Also it should be clear that $bigcup limits_{xin U} U_x = U$.



                                                  Now we show that $forall x$ $U_x$ is open.
                                                  Let $yin U_x subseteq U$.
                                                  Since $U$ is open there exists $epsilon>0$ such that $(y-epsilon, y+epsilon )subseteq U$.
                                                  Sets of real numbers are connected iff they are intervals, singletons or empty.
                                                  $(y-epsilon,y+epsilon)$ an interval hence it is connected.
                                                  Therefore since $U_y$ is the largest connected subset of $U$ containing $y$ we must have $(y-epsilon,y+epsilon)subseteq U_y =U_x$.
                                                  This shows that $U_x$ is open for all $x$.



                                                  $U_x$ open and connected implies that $U_x$ must be an open interval.



                                                  Also $mathbb{Q}$ dense in $mathbb{R}$,
                                                  so $forall xin U$, $U_xcap mathbb{Q}neq varnothing$ and $U_x=U_q$ for some $qinmathbb{Q}$.
                                                  So we can write ${U_x}_{xin U}={U_q}_{qin S}$ for some $Ssubseteq mathbb{Q}$.
                                                  $mathbb{Q}$ is countable so $S$ is at most countable.



                                                  In conclusion, we have just shown that the union of the connected components of $U$ is a disjoint union of open intervals that equals $U$ and is at most countable.






                                                  share|cite|improve this answer











                                                  $endgroup$
















                                                    2












                                                    2








                                                    2





                                                    $begingroup$

                                                    Essentially nothing differs here from the two previous responses which rely principally on the fact that $mathbb{R} $ is locally connected. However I present here a proof that hopefully will feel accessible to readers with a slightly lower level of topological literary at the cost of appearing cumbersome to an expert.



                                                    Consider the connected components of $U$.
                                                    $U_x subseteq U$ is defined to be the connected component of U containing $x$ if $U_x$ is the largest connected subset of $U$ which contains $x$.
                                                    Clearly by definition $U_x=U_v$ if $v in U_x$.
                                                    Therefore if $U_a cap U_b neq varnothing$ then $U_a=U_b$.
                                                    We see that ${U_x}_{xin U}$ is a disjoint collection.
                                                    Also it should be clear that $bigcup limits_{xin U} U_x = U$.



                                                    Now we show that $forall x$ $U_x$ is open.
                                                    Let $yin U_x subseteq U$.
                                                    Since $U$ is open there exists $epsilon>0$ such that $(y-epsilon, y+epsilon )subseteq U$.
                                                    Sets of real numbers are connected iff they are intervals, singletons or empty.
                                                    $(y-epsilon,y+epsilon)$ an interval hence it is connected.
                                                    Therefore since $U_y$ is the largest connected subset of $U$ containing $y$ we must have $(y-epsilon,y+epsilon)subseteq U_y =U_x$.
                                                    This shows that $U_x$ is open for all $x$.



                                                    $U_x$ open and connected implies that $U_x$ must be an open interval.



                                                    Also $mathbb{Q}$ dense in $mathbb{R}$,
                                                    so $forall xin U$, $U_xcap mathbb{Q}neq varnothing$ and $U_x=U_q$ for some $qinmathbb{Q}$.
                                                    So we can write ${U_x}_{xin U}={U_q}_{qin S}$ for some $Ssubseteq mathbb{Q}$.
                                                    $mathbb{Q}$ is countable so $S$ is at most countable.



                                                    In conclusion, we have just shown that the union of the connected components of $U$ is a disjoint union of open intervals that equals $U$ and is at most countable.






                                                    share|cite|improve this answer











                                                    $endgroup$



                                                    Essentially nothing differs here from the two previous responses which rely principally on the fact that $mathbb{R} $ is locally connected. However I present here a proof that hopefully will feel accessible to readers with a slightly lower level of topological literary at the cost of appearing cumbersome to an expert.



                                                    Consider the connected components of $U$.
                                                    $U_x subseteq U$ is defined to be the connected component of U containing $x$ if $U_x$ is the largest connected subset of $U$ which contains $x$.
                                                    Clearly by definition $U_x=U_v$ if $v in U_x$.
                                                    Therefore if $U_a cap U_b neq varnothing$ then $U_a=U_b$.
                                                    We see that ${U_x}_{xin U}$ is a disjoint collection.
                                                    Also it should be clear that $bigcup limits_{xin U} U_x = U$.



                                                    Now we show that $forall x$ $U_x$ is open.
                                                    Let $yin U_x subseteq U$.
                                                    Since $U$ is open there exists $epsilon>0$ such that $(y-epsilon, y+epsilon )subseteq U$.
                                                    Sets of real numbers are connected iff they are intervals, singletons or empty.
                                                    $(y-epsilon,y+epsilon)$ an interval hence it is connected.
                                                    Therefore since $U_y$ is the largest connected subset of $U$ containing $y$ we must have $(y-epsilon,y+epsilon)subseteq U_y =U_x$.
                                                    This shows that $U_x$ is open for all $x$.



                                                    $U_x$ open and connected implies that $U_x$ must be an open interval.



                                                    Also $mathbb{Q}$ dense in $mathbb{R}$,
                                                    so $forall xin U$, $U_xcap mathbb{Q}neq varnothing$ and $U_x=U_q$ for some $qinmathbb{Q}$.
                                                    So we can write ${U_x}_{xin U}={U_q}_{qin S}$ for some $Ssubseteq mathbb{Q}$.
                                                    $mathbb{Q}$ is countable so $S$ is at most countable.



                                                    In conclusion, we have just shown that the union of the connected components of $U$ is a disjoint union of open intervals that equals $U$ and is at most countable.







                                                    share|cite|improve this answer














                                                    share|cite|improve this answer



                                                    share|cite|improve this answer








                                                    answered May 31 '17 at 22:25


























                                                    community wiki





                                                    Nathan A.S.
























                                                        1












                                                        $begingroup$

                                                        The proof that every open set is a disjoint union of countably many open intervals relies on three facts:




                                                        • $Bbb R$ is locally-connected

                                                        • $Bbb R$ is ccc

                                                        • The open connected sets in $Bbb R$ are open intervals


                                                        Let $Usubseteq Bbb R$ be open. Then there is a collection of disjoint, open, connected sets ${G_alpha}_{alphain A}$ such that $U=bigcup_{alphain A} G_alpha$. Since $Bbb R$ is ccc, the collection ${G_alpha}$ is at most countable. Since the open connected sets $Bbb R$ are open intervals, ${G_alpha}$ is a countable collection of disjoint, open intervals.



                                                        The first two facts allow us to see some generalizations. Namely any open set in a locally-connected, ccc space is a countable disjoint union of connected open sets. This applies to any Euclidean space. Although open connected subsets of Euclidean space are more complicated than open intervals, they are still relatively well-behaved.






                                                        share|cite|improve this answer











                                                        $endgroup$













                                                        • $begingroup$
                                                          What does "R is ccc" mean?
                                                          $endgroup$
                                                          – Christian Bueno
                                                          Feb 24 '15 at 8:39










                                                        • $begingroup$
                                                          It means "satisfies the countable-chain condition" which really concerns itself with anti-chains. It means that every collection of disjoint open sets is at most countable. Every separable space is ccc.
                                                          $endgroup$
                                                          – Robert Wolfe
                                                          Feb 25 '15 at 0:56










                                                        • $begingroup$
                                                          Ok thanks for clearing that up.
                                                          $endgroup$
                                                          – Christian Bueno
                                                          Feb 25 '15 at 5:18










                                                        • $begingroup$
                                                          how to decompose (0,1} into a countable union of disjoint open intervals?
                                                          $endgroup$
                                                          – Bear and bunny
                                                          Mar 27 '15 at 15:31
















                                                        1












                                                        $begingroup$

                                                        The proof that every open set is a disjoint union of countably many open intervals relies on three facts:




                                                        • $Bbb R$ is locally-connected

                                                        • $Bbb R$ is ccc

                                                        • The open connected sets in $Bbb R$ are open intervals


                                                        Let $Usubseteq Bbb R$ be open. Then there is a collection of disjoint, open, connected sets ${G_alpha}_{alphain A}$ such that $U=bigcup_{alphain A} G_alpha$. Since $Bbb R$ is ccc, the collection ${G_alpha}$ is at most countable. Since the open connected sets $Bbb R$ are open intervals, ${G_alpha}$ is a countable collection of disjoint, open intervals.



                                                        The first two facts allow us to see some generalizations. Namely any open set in a locally-connected, ccc space is a countable disjoint union of connected open sets. This applies to any Euclidean space. Although open connected subsets of Euclidean space are more complicated than open intervals, they are still relatively well-behaved.






                                                        share|cite|improve this answer











                                                        $endgroup$













                                                        • $begingroup$
                                                          What does "R is ccc" mean?
                                                          $endgroup$
                                                          – Christian Bueno
                                                          Feb 24 '15 at 8:39










                                                        • $begingroup$
                                                          It means "satisfies the countable-chain condition" which really concerns itself with anti-chains. It means that every collection of disjoint open sets is at most countable. Every separable space is ccc.
                                                          $endgroup$
                                                          – Robert Wolfe
                                                          Feb 25 '15 at 0:56










                                                        • $begingroup$
                                                          Ok thanks for clearing that up.
                                                          $endgroup$
                                                          – Christian Bueno
                                                          Feb 25 '15 at 5:18










                                                        • $begingroup$
                                                          how to decompose (0,1} into a countable union of disjoint open intervals?
                                                          $endgroup$
                                                          – Bear and bunny
                                                          Mar 27 '15 at 15:31














                                                        1












                                                        1








                                                        1





                                                        $begingroup$

                                                        The proof that every open set is a disjoint union of countably many open intervals relies on three facts:




                                                        • $Bbb R$ is locally-connected

                                                        • $Bbb R$ is ccc

                                                        • The open connected sets in $Bbb R$ are open intervals


                                                        Let $Usubseteq Bbb R$ be open. Then there is a collection of disjoint, open, connected sets ${G_alpha}_{alphain A}$ such that $U=bigcup_{alphain A} G_alpha$. Since $Bbb R$ is ccc, the collection ${G_alpha}$ is at most countable. Since the open connected sets $Bbb R$ are open intervals, ${G_alpha}$ is a countable collection of disjoint, open intervals.



                                                        The first two facts allow us to see some generalizations. Namely any open set in a locally-connected, ccc space is a countable disjoint union of connected open sets. This applies to any Euclidean space. Although open connected subsets of Euclidean space are more complicated than open intervals, they are still relatively well-behaved.






                                                        share|cite|improve this answer











                                                        $endgroup$



                                                        The proof that every open set is a disjoint union of countably many open intervals relies on three facts:




                                                        • $Bbb R$ is locally-connected

                                                        • $Bbb R$ is ccc

                                                        • The open connected sets in $Bbb R$ are open intervals


                                                        Let $Usubseteq Bbb R$ be open. Then there is a collection of disjoint, open, connected sets ${G_alpha}_{alphain A}$ such that $U=bigcup_{alphain A} G_alpha$. Since $Bbb R$ is ccc, the collection ${G_alpha}$ is at most countable. Since the open connected sets $Bbb R$ are open intervals, ${G_alpha}$ is a countable collection of disjoint, open intervals.



                                                        The first two facts allow us to see some generalizations. Namely any open set in a locally-connected, ccc space is a countable disjoint union of connected open sets. This applies to any Euclidean space. Although open connected subsets of Euclidean space are more complicated than open intervals, they are still relatively well-behaved.







                                                        share|cite|improve this answer














                                                        share|cite|improve this answer



                                                        share|cite|improve this answer








                                                        answered Jul 31 '14 at 18:16


























                                                        community wiki





                                                        Robert Wolfe













                                                        • $begingroup$
                                                          What does "R is ccc" mean?
                                                          $endgroup$
                                                          – Christian Bueno
                                                          Feb 24 '15 at 8:39










                                                        • $begingroup$
                                                          It means "satisfies the countable-chain condition" which really concerns itself with anti-chains. It means that every collection of disjoint open sets is at most countable. Every separable space is ccc.
                                                          $endgroup$
                                                          – Robert Wolfe
                                                          Feb 25 '15 at 0:56










                                                        • $begingroup$
                                                          Ok thanks for clearing that up.
                                                          $endgroup$
                                                          – Christian Bueno
                                                          Feb 25 '15 at 5:18










                                                        • $begingroup$
                                                          how to decompose (0,1} into a countable union of disjoint open intervals?
                                                          $endgroup$
                                                          – Bear and bunny
                                                          Mar 27 '15 at 15:31


















                                                        • $begingroup$
                                                          What does "R is ccc" mean?
                                                          $endgroup$
                                                          – Christian Bueno
                                                          Feb 24 '15 at 8:39










                                                        • $begingroup$
                                                          It means "satisfies the countable-chain condition" which really concerns itself with anti-chains. It means that every collection of disjoint open sets is at most countable. Every separable space is ccc.
                                                          $endgroup$
                                                          – Robert Wolfe
                                                          Feb 25 '15 at 0:56










                                                        • $begingroup$
                                                          Ok thanks for clearing that up.
                                                          $endgroup$
                                                          – Christian Bueno
                                                          Feb 25 '15 at 5:18










                                                        • $begingroup$
                                                          how to decompose (0,1} into a countable union of disjoint open intervals?
                                                          $endgroup$
                                                          – Bear and bunny
                                                          Mar 27 '15 at 15:31
















                                                        $begingroup$
                                                        What does "R is ccc" mean?
                                                        $endgroup$
                                                        – Christian Bueno
                                                        Feb 24 '15 at 8:39




                                                        $begingroup$
                                                        What does "R is ccc" mean?
                                                        $endgroup$
                                                        – Christian Bueno
                                                        Feb 24 '15 at 8:39












                                                        $begingroup$
                                                        It means "satisfies the countable-chain condition" which really concerns itself with anti-chains. It means that every collection of disjoint open sets is at most countable. Every separable space is ccc.
                                                        $endgroup$
                                                        – Robert Wolfe
                                                        Feb 25 '15 at 0:56




                                                        $begingroup$
                                                        It means "satisfies the countable-chain condition" which really concerns itself with anti-chains. It means that every collection of disjoint open sets is at most countable. Every separable space is ccc.
                                                        $endgroup$
                                                        – Robert Wolfe
                                                        Feb 25 '15 at 0:56












                                                        $begingroup$
                                                        Ok thanks for clearing that up.
                                                        $endgroup$
                                                        – Christian Bueno
                                                        Feb 25 '15 at 5:18




                                                        $begingroup$
                                                        Ok thanks for clearing that up.
                                                        $endgroup$
                                                        – Christian Bueno
                                                        Feb 25 '15 at 5:18












                                                        $begingroup$
                                                        how to decompose (0,1} into a countable union of disjoint open intervals?
                                                        $endgroup$
                                                        – Bear and bunny
                                                        Mar 27 '15 at 15:31




                                                        $begingroup$
                                                        how to decompose (0,1} into a countable union of disjoint open intervals?
                                                        $endgroup$
                                                        – Bear and bunny
                                                        Mar 27 '15 at 15:31











                                                        1












                                                        $begingroup$

                                                        The following is certainly not the quickest approach to a proof, but when this question was first posed to me in class, my first intuition was to use some elementary graph theory:





                                                        Let $U$ be an open set of $mathbb{R}$. As we know, $mathbb{R}$ has a countable basis $mathcal{B}$ comprised of connected open sets and so we may write $U=bigcup_{nin I} U_n$, where for each $n$ we have $U_ninmathcal{B}$ and $I$ is some countable index set.



                                                        Let $G$ be the intersection graph of ${U_n}$. That is to say, the vertex set of $G$ is simply ${U_n}$ and there is an edge between $U_i$ and $U_j$ iff they have nonempty intersection. It's easy to convince yourself that:




                                                        • This graph must have countably many graphically-connected components (otherwise we'd have uncountably many vertices which is impossible).

                                                        • The intersection graph of $Asubseteq{U_n}$ is graphically-connected iff for any two $V,Win A$ there is a sequence $V=U_{n_1},U_{n_2},ldots,U_{n_k}=W$ such that $U_{n_i}cap U_{n_{i+1}}neqvarnothing$.

                                                        • The union $bigcup A$ is a connected set of $mathbb{R}$ whenever the intersection graph of $A$ is graphically-connected.


                                                        Thus, when we take the union of all the vertices within a graphically-connected component, for every component, we obtain countably-many connected open sets. The union of these sets is of course $U$ itself. Since the connected open sets of $mathbb{R}$ are intervals (including rays), we're done.





                                                        Side Note: This would also work in $mathbb{R}^n$ or in general, any topological space $X$ that has a countable basis comprised of connected sets. Well, so long as we replace countable union of disjoint open intervals with countable union of disjoint open connected sets.






                                                        share|cite|improve this answer











                                                        $endgroup$


















                                                          1












                                                          $begingroup$

                                                          The following is certainly not the quickest approach to a proof, but when this question was first posed to me in class, my first intuition was to use some elementary graph theory:





                                                          Let $U$ be an open set of $mathbb{R}$. As we know, $mathbb{R}$ has a countable basis $mathcal{B}$ comprised of connected open sets and so we may write $U=bigcup_{nin I} U_n$, where for each $n$ we have $U_ninmathcal{B}$ and $I$ is some countable index set.



                                                          Let $G$ be the intersection graph of ${U_n}$. That is to say, the vertex set of $G$ is simply ${U_n}$ and there is an edge between $U_i$ and $U_j$ iff they have nonempty intersection. It's easy to convince yourself that:




                                                          • This graph must have countably many graphically-connected components (otherwise we'd have uncountably many vertices which is impossible).

                                                          • The intersection graph of $Asubseteq{U_n}$ is graphically-connected iff for any two $V,Win A$ there is a sequence $V=U_{n_1},U_{n_2},ldots,U_{n_k}=W$ such that $U_{n_i}cap U_{n_{i+1}}neqvarnothing$.

                                                          • The union $bigcup A$ is a connected set of $mathbb{R}$ whenever the intersection graph of $A$ is graphically-connected.


                                                          Thus, when we take the union of all the vertices within a graphically-connected component, for every component, we obtain countably-many connected open sets. The union of these sets is of course $U$ itself. Since the connected open sets of $mathbb{R}$ are intervals (including rays), we're done.





                                                          Side Note: This would also work in $mathbb{R}^n$ or in general, any topological space $X$ that has a countable basis comprised of connected sets. Well, so long as we replace countable union of disjoint open intervals with countable union of disjoint open connected sets.






                                                          share|cite|improve this answer











                                                          $endgroup$
















                                                            1












                                                            1








                                                            1





                                                            $begingroup$

                                                            The following is certainly not the quickest approach to a proof, but when this question was first posed to me in class, my first intuition was to use some elementary graph theory:





                                                            Let $U$ be an open set of $mathbb{R}$. As we know, $mathbb{R}$ has a countable basis $mathcal{B}$ comprised of connected open sets and so we may write $U=bigcup_{nin I} U_n$, where for each $n$ we have $U_ninmathcal{B}$ and $I$ is some countable index set.



                                                            Let $G$ be the intersection graph of ${U_n}$. That is to say, the vertex set of $G$ is simply ${U_n}$ and there is an edge between $U_i$ and $U_j$ iff they have nonempty intersection. It's easy to convince yourself that:




                                                            • This graph must have countably many graphically-connected components (otherwise we'd have uncountably many vertices which is impossible).

                                                            • The intersection graph of $Asubseteq{U_n}$ is graphically-connected iff for any two $V,Win A$ there is a sequence $V=U_{n_1},U_{n_2},ldots,U_{n_k}=W$ such that $U_{n_i}cap U_{n_{i+1}}neqvarnothing$.

                                                            • The union $bigcup A$ is a connected set of $mathbb{R}$ whenever the intersection graph of $A$ is graphically-connected.


                                                            Thus, when we take the union of all the vertices within a graphically-connected component, for every component, we obtain countably-many connected open sets. The union of these sets is of course $U$ itself. Since the connected open sets of $mathbb{R}$ are intervals (including rays), we're done.





                                                            Side Note: This would also work in $mathbb{R}^n$ or in general, any topological space $X$ that has a countable basis comprised of connected sets. Well, so long as we replace countable union of disjoint open intervals with countable union of disjoint open connected sets.






                                                            share|cite|improve this answer











                                                            $endgroup$



                                                            The following is certainly not the quickest approach to a proof, but when this question was first posed to me in class, my first intuition was to use some elementary graph theory:





                                                            Let $U$ be an open set of $mathbb{R}$. As we know, $mathbb{R}$ has a countable basis $mathcal{B}$ comprised of connected open sets and so we may write $U=bigcup_{nin I} U_n$, where for each $n$ we have $U_ninmathcal{B}$ and $I$ is some countable index set.



                                                            Let $G$ be the intersection graph of ${U_n}$. That is to say, the vertex set of $G$ is simply ${U_n}$ and there is an edge between $U_i$ and $U_j$ iff they have nonempty intersection. It's easy to convince yourself that:




                                                            • This graph must have countably many graphically-connected components (otherwise we'd have uncountably many vertices which is impossible).

                                                            • The intersection graph of $Asubseteq{U_n}$ is graphically-connected iff for any two $V,Win A$ there is a sequence $V=U_{n_1},U_{n_2},ldots,U_{n_k}=W$ such that $U_{n_i}cap U_{n_{i+1}}neqvarnothing$.

                                                            • The union $bigcup A$ is a connected set of $mathbb{R}$ whenever the intersection graph of $A$ is graphically-connected.


                                                            Thus, when we take the union of all the vertices within a graphically-connected component, for every component, we obtain countably-many connected open sets. The union of these sets is of course $U$ itself. Since the connected open sets of $mathbb{R}$ are intervals (including rays), we're done.





                                                            Side Note: This would also work in $mathbb{R}^n$ or in general, any topological space $X$ that has a countable basis comprised of connected sets. Well, so long as we replace countable union of disjoint open intervals with countable union of disjoint open connected sets.







                                                            share|cite|improve this answer














                                                            share|cite|improve this answer



                                                            share|cite|improve this answer








                                                            answered Feb 24 '15 at 8:38


























                                                            community wiki





                                                            Christian Bueno
























                                                                1












                                                                $begingroup$

                                                                The balls with radii $frac{1}{n}$ and center at a rational number form a basis for the euclidean topology. This family is countable since $mathbb N times mathbb Q equiv mathbb N$ and we have a countable basis $(B_lambda)_{ , lambda in mathbb N times mathbb Q}$ of open intervals for $mathbb R$.



                                                                Let $U$ be a nonempty open set in $mathbb R$; we can express it as countable union of open balls from $(B_lambda)$.
                                                                Also,
                                                                $tag 1 text{ }$
                                                                $quad$ If two open intervals have a nonempty intersection, then their union is also an open interval.



                                                                So if $U$ is a finite union of the $B_lambda$, it is an easy matter to combine the $B_lambda$, if necessary, and writing $U$ as a disjoint union of a finite number of open intervals. So assume, WLOG, that



                                                                $tag 2 U = bigcup_{, n in mathbb N ,} B_n$.



                                                                We define a relation on our (new) index set $mathbb N$ with $msim n$ if $B_m cap B_n ne emptyset$ or there is is a finite 'nonempty intersection $Btext{-}$chain' connecting $B_m$ with $B_n$. It is easy to see that this partitions $mathbb N$ and that taking the corresponding unions of the $B_n$ over an index $lambda text{-}$ block gives a partition of $U$. Also, using (1), we can show that we have also expressed $U$ as a countable union of disjoint open intervals.






                                                                share|cite|improve this answer











                                                                $endgroup$


















                                                                  1












                                                                  $begingroup$

                                                                  The balls with radii $frac{1}{n}$ and center at a rational number form a basis for the euclidean topology. This family is countable since $mathbb N times mathbb Q equiv mathbb N$ and we have a countable basis $(B_lambda)_{ , lambda in mathbb N times mathbb Q}$ of open intervals for $mathbb R$.



                                                                  Let $U$ be a nonempty open set in $mathbb R$; we can express it as countable union of open balls from $(B_lambda)$.
                                                                  Also,
                                                                  $tag 1 text{ }$
                                                                  $quad$ If two open intervals have a nonempty intersection, then their union is also an open interval.



                                                                  So if $U$ is a finite union of the $B_lambda$, it is an easy matter to combine the $B_lambda$, if necessary, and writing $U$ as a disjoint union of a finite number of open intervals. So assume, WLOG, that



                                                                  $tag 2 U = bigcup_{, n in mathbb N ,} B_n$.



                                                                  We define a relation on our (new) index set $mathbb N$ with $msim n$ if $B_m cap B_n ne emptyset$ or there is is a finite 'nonempty intersection $Btext{-}$chain' connecting $B_m$ with $B_n$. It is easy to see that this partitions $mathbb N$ and that taking the corresponding unions of the $B_n$ over an index $lambda text{-}$ block gives a partition of $U$. Also, using (1), we can show that we have also expressed $U$ as a countable union of disjoint open intervals.






                                                                  share|cite|improve this answer











                                                                  $endgroup$
















                                                                    1












                                                                    1








                                                                    1





                                                                    $begingroup$

                                                                    The balls with radii $frac{1}{n}$ and center at a rational number form a basis for the euclidean topology. This family is countable since $mathbb N times mathbb Q equiv mathbb N$ and we have a countable basis $(B_lambda)_{ , lambda in mathbb N times mathbb Q}$ of open intervals for $mathbb R$.



                                                                    Let $U$ be a nonempty open set in $mathbb R$; we can express it as countable union of open balls from $(B_lambda)$.
                                                                    Also,
                                                                    $tag 1 text{ }$
                                                                    $quad$ If two open intervals have a nonempty intersection, then their union is also an open interval.



                                                                    So if $U$ is a finite union of the $B_lambda$, it is an easy matter to combine the $B_lambda$, if necessary, and writing $U$ as a disjoint union of a finite number of open intervals. So assume, WLOG, that



                                                                    $tag 2 U = bigcup_{, n in mathbb N ,} B_n$.



                                                                    We define a relation on our (new) index set $mathbb N$ with $msim n$ if $B_m cap B_n ne emptyset$ or there is is a finite 'nonempty intersection $Btext{-}$chain' connecting $B_m$ with $B_n$. It is easy to see that this partitions $mathbb N$ and that taking the corresponding unions of the $B_n$ over an index $lambda text{-}$ block gives a partition of $U$. Also, using (1), we can show that we have also expressed $U$ as a countable union of disjoint open intervals.






                                                                    share|cite|improve this answer











                                                                    $endgroup$



                                                                    The balls with radii $frac{1}{n}$ and center at a rational number form a basis for the euclidean topology. This family is countable since $mathbb N times mathbb Q equiv mathbb N$ and we have a countable basis $(B_lambda)_{ , lambda in mathbb N times mathbb Q}$ of open intervals for $mathbb R$.



                                                                    Let $U$ be a nonempty open set in $mathbb R$; we can express it as countable union of open balls from $(B_lambda)$.
                                                                    Also,
                                                                    $tag 1 text{ }$
                                                                    $quad$ If two open intervals have a nonempty intersection, then their union is also an open interval.



                                                                    So if $U$ is a finite union of the $B_lambda$, it is an easy matter to combine the $B_lambda$, if necessary, and writing $U$ as a disjoint union of a finite number of open intervals. So assume, WLOG, that



                                                                    $tag 2 U = bigcup_{, n in mathbb N ,} B_n$.



                                                                    We define a relation on our (new) index set $mathbb N$ with $msim n$ if $B_m cap B_n ne emptyset$ or there is is a finite 'nonempty intersection $Btext{-}$chain' connecting $B_m$ with $B_n$. It is easy to see that this partitions $mathbb N$ and that taking the corresponding unions of the $B_n$ over an index $lambda text{-}$ block gives a partition of $U$. Also, using (1), we can show that we have also expressed $U$ as a countable union of disjoint open intervals.







                                                                    share|cite|improve this answer














                                                                    share|cite|improve this answer



                                                                    share|cite|improve this answer








                                                                    edited Nov 6 '17 at 19:32


























                                                                    community wiki





                                                                    3 revs
                                                                    MikeMathMan
























                                                                        1












                                                                        $begingroup$

                                                                        More of a question than answer. I am a chemist turned pharmacist, who wishes to have studied mathematics. I am trying to work through Rudin's Principles of Mathematical Analysis. I envy you all who are involved in math for a career.



                                                                        Can someone give me feedback on my attempt at a proof in $mathbb R? Completely novice and not at all pretty, but is it sound?



                                                                        segment := open interval in R.



                                                                        Lemma: disjoint segments in R are separated (proof not shown).



                                                                        It follows from the lemma that an open connected subset of R cannot be the union of disjoint segments.



                                                                        Let E be an open subset of R. Since R is separable by Rudin prob 2.22, there exists a subset, D, of R that is countable and dense in R. Assume E is connected, which includes E=R, then E is the union of an at most countable collection of open segments, containing only E.



                                                                        Suppose E is separated. Then E is the union of a collection of disjoint segments, including the possibility of segments unbounded above or below.



                                                                        If the collection is finite, then it is at most countable.



                                                                        Assume the collection of segments is infinite. Because D is dense in R and E is contained in R, every open subset of E contains a point of D. Then each of the infinitely many disjoint segments contains a unique point of D. Since D is countable, a one-to-one correspondence between a unique point of D and the segment is established. This implies that there are countably many disjoint segments in the collection.



                                                                        Therefore E is the union of finitely many or countably many
                                                                        , hence at most countably many, disjoint segments.






                                                                        share|cite|improve this answer











                                                                        $endgroup$









                                                                        • 1




                                                                          $begingroup$
                                                                          More of a question than answer. I suppose this would be better posted as a question than as an answer, then. Suppose E is separated. Then E is the union of a collection of disjoint segments. It's not obvious (to me, at least) where this follows from in the given context.
                                                                          $endgroup$
                                                                          – dxiv
                                                                          Oct 9 '16 at 5:43








                                                                        • 1




                                                                          $begingroup$
                                                                          I know. Just learning how things work on this site. Which I think is quite amazing.
                                                                          $endgroup$
                                                                          – RJM
                                                                          Oct 9 '16 at 5:48










                                                                        • $begingroup$
                                                                          @dxiv Since E separated (or not connected), would it have to at least be the union of 2 separated sets. Since it is open, they have to be open subsets, which implies open interval in R1. Thank you!
                                                                          $endgroup$
                                                                          – RJM
                                                                          Oct 9 '16 at 5:53












                                                                        • $begingroup$
                                                                          @dxiv i will not keep bugging you. Any tips on how to become mathematician-like on one's own in essentially matgematica isolation?
                                                                          $endgroup$
                                                                          – RJM
                                                                          Oct 9 '16 at 6:00










                                                                        • $begingroup$
                                                                          IMHO that latter implies open interval in R1 is a part that needs to be proved in this context. See for example Intervals are connected and the only connected sets in R. Anyway, as I said, yours appears to be better suited as a question than as an answer. P.S. Keep plugging away until it all starts making sense ;-)
                                                                          $endgroup$
                                                                          – dxiv
                                                                          Oct 9 '16 at 6:00


















                                                                        1












                                                                        $begingroup$

                                                                        More of a question than answer. I am a chemist turned pharmacist, who wishes to have studied mathematics. I am trying to work through Rudin's Principles of Mathematical Analysis. I envy you all who are involved in math for a career.



                                                                        Can someone give me feedback on my attempt at a proof in $mathbb R? Completely novice and not at all pretty, but is it sound?



                                                                        segment := open interval in R.



                                                                        Lemma: disjoint segments in R are separated (proof not shown).



                                                                        It follows from the lemma that an open connected subset of R cannot be the union of disjoint segments.



                                                                        Let E be an open subset of R. Since R is separable by Rudin prob 2.22, there exists a subset, D, of R that is countable and dense in R. Assume E is connected, which includes E=R, then E is the union of an at most countable collection of open segments, containing only E.



                                                                        Suppose E is separated. Then E is the union of a collection of disjoint segments, including the possibility of segments unbounded above or below.



                                                                        If the collection is finite, then it is at most countable.



                                                                        Assume the collection of segments is infinite. Because D is dense in R and E is contained in R, every open subset of E contains a point of D. Then each of the infinitely many disjoint segments contains a unique point of D. Since D is countable, a one-to-one correspondence between a unique point of D and the segment is established. This implies that there are countably many disjoint segments in the collection.



                                                                        Therefore E is the union of finitely many or countably many
                                                                        , hence at most countably many, disjoint segments.






                                                                        share|cite|improve this answer











                                                                        $endgroup$









                                                                        • 1




                                                                          $begingroup$
                                                                          More of a question than answer. I suppose this would be better posted as a question than as an answer, then. Suppose E is separated. Then E is the union of a collection of disjoint segments. It's not obvious (to me, at least) where this follows from in the given context.
                                                                          $endgroup$
                                                                          – dxiv
                                                                          Oct 9 '16 at 5:43








                                                                        • 1




                                                                          $begingroup$
                                                                          I know. Just learning how things work on this site. Which I think is quite amazing.
                                                                          $endgroup$
                                                                          – RJM
                                                                          Oct 9 '16 at 5:48










                                                                        • $begingroup$
                                                                          @dxiv Since E separated (or not connected), would it have to at least be the union of 2 separated sets. Since it is open, they have to be open subsets, which implies open interval in R1. Thank you!
                                                                          $endgroup$
                                                                          – RJM
                                                                          Oct 9 '16 at 5:53












                                                                        • $begingroup$
                                                                          @dxiv i will not keep bugging you. Any tips on how to become mathematician-like on one's own in essentially matgematica isolation?
                                                                          $endgroup$
                                                                          – RJM
                                                                          Oct 9 '16 at 6:00










                                                                        • $begingroup$
                                                                          IMHO that latter implies open interval in R1 is a part that needs to be proved in this context. See for example Intervals are connected and the only connected sets in R. Anyway, as I said, yours appears to be better suited as a question than as an answer. P.S. Keep plugging away until it all starts making sense ;-)
                                                                          $endgroup$
                                                                          – dxiv
                                                                          Oct 9 '16 at 6:00
















                                                                        1












                                                                        1








                                                                        1





                                                                        $begingroup$

                                                                        More of a question than answer. I am a chemist turned pharmacist, who wishes to have studied mathematics. I am trying to work through Rudin's Principles of Mathematical Analysis. I envy you all who are involved in math for a career.



                                                                        Can someone give me feedback on my attempt at a proof in $mathbb R? Completely novice and not at all pretty, but is it sound?



                                                                        segment := open interval in R.



                                                                        Lemma: disjoint segments in R are separated (proof not shown).



                                                                        It follows from the lemma that an open connected subset of R cannot be the union of disjoint segments.



                                                                        Let E be an open subset of R. Since R is separable by Rudin prob 2.22, there exists a subset, D, of R that is countable and dense in R. Assume E is connected, which includes E=R, then E is the union of an at most countable collection of open segments, containing only E.



                                                                        Suppose E is separated. Then E is the union of a collection of disjoint segments, including the possibility of segments unbounded above or below.



                                                                        If the collection is finite, then it is at most countable.



                                                                        Assume the collection of segments is infinite. Because D is dense in R and E is contained in R, every open subset of E contains a point of D. Then each of the infinitely many disjoint segments contains a unique point of D. Since D is countable, a one-to-one correspondence between a unique point of D and the segment is established. This implies that there are countably many disjoint segments in the collection.



                                                                        Therefore E is the union of finitely many or countably many
                                                                        , hence at most countably many, disjoint segments.






                                                                        share|cite|improve this answer











                                                                        $endgroup$



                                                                        More of a question than answer. I am a chemist turned pharmacist, who wishes to have studied mathematics. I am trying to work through Rudin's Principles of Mathematical Analysis. I envy you all who are involved in math for a career.



                                                                        Can someone give me feedback on my attempt at a proof in $mathbb R? Completely novice and not at all pretty, but is it sound?



                                                                        segment := open interval in R.



                                                                        Lemma: disjoint segments in R are separated (proof not shown).



                                                                        It follows from the lemma that an open connected subset of R cannot be the union of disjoint segments.



                                                                        Let E be an open subset of R. Since R is separable by Rudin prob 2.22, there exists a subset, D, of R that is countable and dense in R. Assume E is connected, which includes E=R, then E is the union of an at most countable collection of open segments, containing only E.



                                                                        Suppose E is separated. Then E is the union of a collection of disjoint segments, including the possibility of segments unbounded above or below.



                                                                        If the collection is finite, then it is at most countable.



                                                                        Assume the collection of segments is infinite. Because D is dense in R and E is contained in R, every open subset of E contains a point of D. Then each of the infinitely many disjoint segments contains a unique point of D. Since D is countable, a one-to-one correspondence between a unique point of D and the segment is established. This implies that there are countably many disjoint segments in the collection.



                                                                        Therefore E is the union of finitely many or countably many
                                                                        , hence at most countably many, disjoint segments.







                                                                        share|cite|improve this answer














                                                                        share|cite|improve this answer



                                                                        share|cite|improve this answer








                                                                        edited Sep 24 '18 at 17:43


























                                                                        community wiki





                                                                        3 revs, 2 users 97%
                                                                        RJM









                                                                        • 1




                                                                          $begingroup$
                                                                          More of a question than answer. I suppose this would be better posted as a question than as an answer, then. Suppose E is separated. Then E is the union of a collection of disjoint segments. It's not obvious (to me, at least) where this follows from in the given context.
                                                                          $endgroup$
                                                                          – dxiv
                                                                          Oct 9 '16 at 5:43








                                                                        • 1




                                                                          $begingroup$
                                                                          I know. Just learning how things work on this site. Which I think is quite amazing.
                                                                          $endgroup$
                                                                          – RJM
                                                                          Oct 9 '16 at 5:48










                                                                        • $begingroup$
                                                                          @dxiv Since E separated (or not connected), would it have to at least be the union of 2 separated sets. Since it is open, they have to be open subsets, which implies open interval in R1. Thank you!
                                                                          $endgroup$
                                                                          – RJM
                                                                          Oct 9 '16 at 5:53












                                                                        • $begingroup$
                                                                          @dxiv i will not keep bugging you. Any tips on how to become mathematician-like on one's own in essentially matgematica isolation?
                                                                          $endgroup$
                                                                          – RJM
                                                                          Oct 9 '16 at 6:00










                                                                        • $begingroup$
                                                                          IMHO that latter implies open interval in R1 is a part that needs to be proved in this context. See for example Intervals are connected and the only connected sets in R. Anyway, as I said, yours appears to be better suited as a question than as an answer. P.S. Keep plugging away until it all starts making sense ;-)
                                                                          $endgroup$
                                                                          – dxiv
                                                                          Oct 9 '16 at 6:00
















                                                                        • 1




                                                                          $begingroup$
                                                                          More of a question than answer. I suppose this would be better posted as a question than as an answer, then. Suppose E is separated. Then E is the union of a collection of disjoint segments. It's not obvious (to me, at least) where this follows from in the given context.
                                                                          $endgroup$
                                                                          – dxiv
                                                                          Oct 9 '16 at 5:43








                                                                        • 1




                                                                          $begingroup$
                                                                          I know. Just learning how things work on this site. Which I think is quite amazing.
                                                                          $endgroup$
                                                                          – RJM
                                                                          Oct 9 '16 at 5:48










                                                                        • $begingroup$
                                                                          @dxiv Since E separated (or not connected), would it have to at least be the union of 2 separated sets. Since it is open, they have to be open subsets, which implies open interval in R1. Thank you!
                                                                          $endgroup$
                                                                          – RJM
                                                                          Oct 9 '16 at 5:53












                                                                        • $begingroup$
                                                                          @dxiv i will not keep bugging you. Any tips on how to become mathematician-like on one's own in essentially matgematica isolation?
                                                                          $endgroup$
                                                                          – RJM
                                                                          Oct 9 '16 at 6:00










                                                                        • $begingroup$
                                                                          IMHO that latter implies open interval in R1 is a part that needs to be proved in this context. See for example Intervals are connected and the only connected sets in R. Anyway, as I said, yours appears to be better suited as a question than as an answer. P.S. Keep plugging away until it all starts making sense ;-)
                                                                          $endgroup$
                                                                          – dxiv
                                                                          Oct 9 '16 at 6:00










                                                                        1




                                                                        1




                                                                        $begingroup$
                                                                        More of a question than answer. I suppose this would be better posted as a question than as an answer, then. Suppose E is separated. Then E is the union of a collection of disjoint segments. It's not obvious (to me, at least) where this follows from in the given context.
                                                                        $endgroup$
                                                                        – dxiv
                                                                        Oct 9 '16 at 5:43






                                                                        $begingroup$
                                                                        More of a question than answer. I suppose this would be better posted as a question than as an answer, then. Suppose E is separated. Then E is the union of a collection of disjoint segments. It's not obvious (to me, at least) where this follows from in the given context.
                                                                        $endgroup$
                                                                        – dxiv
                                                                        Oct 9 '16 at 5:43






                                                                        1




                                                                        1




                                                                        $begingroup$
                                                                        I know. Just learning how things work on this site. Which I think is quite amazing.
                                                                        $endgroup$
                                                                        – RJM
                                                                        Oct 9 '16 at 5:48




                                                                        $begingroup$
                                                                        I know. Just learning how things work on this site. Which I think is quite amazing.
                                                                        $endgroup$
                                                                        – RJM
                                                                        Oct 9 '16 at 5:48












                                                                        $begingroup$
                                                                        @dxiv Since E separated (or not connected), would it have to at least be the union of 2 separated sets. Since it is open, they have to be open subsets, which implies open interval in R1. Thank you!
                                                                        $endgroup$
                                                                        – RJM
                                                                        Oct 9 '16 at 5:53






                                                                        $begingroup$
                                                                        @dxiv Since E separated (or not connected), would it have to at least be the union of 2 separated sets. Since it is open, they have to be open subsets, which implies open interval in R1. Thank you!
                                                                        $endgroup$
                                                                        – RJM
                                                                        Oct 9 '16 at 5:53














                                                                        $begingroup$
                                                                        @dxiv i will not keep bugging you. Any tips on how to become mathematician-like on one's own in essentially matgematica isolation?
                                                                        $endgroup$
                                                                        – RJM
                                                                        Oct 9 '16 at 6:00




                                                                        $begingroup$
                                                                        @dxiv i will not keep bugging you. Any tips on how to become mathematician-like on one's own in essentially matgematica isolation?
                                                                        $endgroup$
                                                                        – RJM
                                                                        Oct 9 '16 at 6:00












                                                                        $begingroup$
                                                                        IMHO that latter implies open interval in R1 is a part that needs to be proved in this context. See for example Intervals are connected and the only connected sets in R. Anyway, as I said, yours appears to be better suited as a question than as an answer. P.S. Keep plugging away until it all starts making sense ;-)
                                                                        $endgroup$
                                                                        – dxiv
                                                                        Oct 9 '16 at 6:00






                                                                        $begingroup$
                                                                        IMHO that latter implies open interval in R1 is a part that needs to be proved in this context. See for example Intervals are connected and the only connected sets in R. Anyway, as I said, yours appears to be better suited as a question than as an answer. P.S. Keep plugging away until it all starts making sense ;-)
                                                                        $endgroup$
                                                                        – dxiv
                                                                        Oct 9 '16 at 6:00




















                                                                        draft saved

                                                                        draft discarded




















































                                                                        Thanks for contributing an answer to Mathematics Stack Exchange!


                                                                        • Please be sure to answer the question. Provide details and share your research!

                                                                        But avoid



                                                                        • Asking for help, clarification, or responding to other answers.

                                                                        • Making statements based on opinion; back them up with references or personal experience.


                                                                        Use MathJax to format equations. MathJax reference.


                                                                        To learn more, see our tips on writing great answers.




                                                                        draft saved


                                                                        draft discarded














                                                                        StackExchange.ready(
                                                                        function () {
                                                                        StackExchange.openid.initPostLogin('.new-post-login', 'https%3a%2f%2fmath.stackexchange.com%2fquestions%2f318299%2fany-open-subset-of-bbb-r-is-a-at-most-countable-union-of-disjoint-open-interv%23new-answer', 'question_page');
                                                                        }
                                                                        );

                                                                        Post as a guest















                                                                        Required, but never shown





















































                                                                        Required, but never shown














                                                                        Required, but never shown












                                                                        Required, but never shown







                                                                        Required, but never shown

































                                                                        Required, but never shown














                                                                        Required, but never shown












                                                                        Required, but never shown







                                                                        Required, but never shown







                                                                        Popular posts from this blog

                                                                        Biblatex bibliography style without URLs when DOI exists (in Overleaf with Zotero bibliography)

                                                                        ComboBox Display Member on multiple fields

                                                                        Is it possible to collect Nectar points via Trainline?